You are on page 1of 158

MENTAL ABILITY INDEX

CLASS-VII
S.No. CHAPTER NAME Pg.No.
1. SERIES 01-06

2. LOGICAL VENN DIAGRAM 07-14

3. NON-VERBAL SERIES 15-18

4. CODING & DECODING 19-26

5. MATHEMATICAL OPERATIONS 27-32

6. ANALYTICAL REASONING 33-38

7. ANALOGY 39-46

8. CLASSIFICATION 47-50

9. NON-VERBAL ANALOGY 51-56

10. NON-VERBAL CLASSIFICATION 57-60

11. DIRECTION SENSE TEST 61-68

www.allenoverseas.com OVERSEAS i
S.No. CHAPTER NAME Pg.No.

12. BLOOD RELATION 69-72

13. CUBE & DICE 73-84

14. NUMBER, RANKING & TIME SEQUENCE TEST 85-90

15. FIGURE FORMATION,CONSTRUCTION OF 91-96


SQUARES & GROUPING OF
IDENTICAL FIGURES
16. INSERTING THE MISSING CHARACTER 97-102

17. MIRROR & WATER IMAGES 103-114

18. ALPHABET TEST 115-122

19. PAPER FOLDING & PAPER CUTTING 123-126

20. ALPHA NUMERIC SEQUENCE PUZZLE & 127-134


LOGICAL SEQUENCE OF WORDS

21. SITTING ARRANGEMENT 135-140

22. DOT SITUATION 141-144

23. SPOTTING OUT THE EMBEDDED FIGURES 145-148

24. COMPLETION OF INCOMPLETE PATTERN 149-152

25. FIGURE MATRIX 153-156

Dear students, we have put in best efforts for making this module highly interesting, easy to understand
BOBA-BB

and free from errors. We welcome your valuable feedback, suggestions or any corrections to further
improve the quality of this module. Kindly mail them to us on overseas.academics@allen.ac.in.

ii OVERSEAS www.allenoverseas.com
Mental Ability

CHAPTER 1 SERIES

A series is a sequence of number/alphabet or both which follow a particular rule. Each element of series is
called term. We have to analyse the pattern find the missing term or next term to continue the pattern.

Types of Series

Number Alphabet Alpha-Numeric Continuous


Series Series Series Pattern
Series
NUMBER SERIES
Number series is a form of number is a certain sequence, where some numbers are missing in that series, we
need to observe first and the find accurate number to that series of numbers.
Solved Examples
Ex-1. Find the missing number in the pattern below.

2 4 6 8 ? 12

(1) 9 (2) 10 (3) 11 (4) 7


2 4 6 8 10 12
Sol.
+2 +2 +2 +2 +2
Hence, the answer is (2).
Ex-2. Choose the correct alternative which replaces the question mark.

6 11 21 36 56 ?

(1) 51 (2) 91 (3) 42 (4) 81

6 11 21 36 56 81
Sol.
+5 +10 +15 +20 +25

\ the next number is 81.


Hence, the answer is (4).
Ex-3. Choose the missing term to complete the given series.
3, 5, 10, 12, 24, 26, 52,?
(1) 104 (2) 102 (3) 54 (4) 50
Sol. We observe that
2nd term=3+2=5
3rd term=5×2=10
4th term = 10+2=12
5th term = 12×2=24
Clearly the pattern is : + 2,× 2 + 2, ×2, +2, ×2, + 2
So, 8th (required) term = 52+2=54
Hence, the answer is (3).

www.allenoverseas.com OVERSEAS 1
Class-VII

ALPHABET SERIES
In this type of questions, a series of single, pairs or groups of letters or combinations of letters are given. The
terms of the series form a certain pattern as regards the position of the letters in the English alphabet. You are
required to decipher this pattern and accordingly find the missing term or the wrong term in the given series.
Solved Examples
Ex-4. Find the next letter in the series.
B D F H J L ?

(1) N (2) P (3) O (4) M


Sol. The answer is N, because the pattern is to count forward in two's from the first given letter.
C E G I K M
B D F H J L N

Hence, the answer is (1).


Ex-5. Find the next pair of letters in the series.

AH BI CJ DK ?

(1) EL (2) PU (3) OT (4) MS


Sol. The answer is EL, because the pattern is to count forwards in ones for both the first letter and the second letter:
First letter ¾® A B C D E
Second letter ¾® H I J K L

AH, BI, CJ, DK, (EL)

Hence, the answer is (1).


Ex-6. Identify the next two alphabets in the sequence given below.

A B B D C F D H E ? ?

(1) E, F (2) F, G (3) F, I (4) J, F


Sol. The given sequence is a combination of two series,
(i) 1st, 3rd, 5th, 7th, 9th, 11th terms i.e., A, B, C, D, E,? and
(ii) 2nd, 4th, 6th, 8th, 10th, terms i.e., B, D, F, H,?
Clearly, (i) consists of consecutive letters while (ii) consists of alternate letters. So, the missing letter in (i) is F, while
that in (ii) is J. Hence, the answer is (4).
ALPHA - NUMERIC SERIES
This type of questions is just a jumbled form of questions of Type I and Type II, which you have just read. Here,
the terms of the given series are a combination of letters and numerals, which move according to a set pattern.
Study the following examples.
Solved Examples
Ex.7 Find the next term in the alpha-numeric series :
Z1A, X2D, V6G, T21J, R88M, P445P,?
(1) N2676S (2) N2676T (3) T2670N (4) T2676N
Sol. Clearly, the patterns followed by the letters are as follows:

1st letter : Z ¾¾¾


–2
® X ¾¾¾
–2
® V ¾¾¾
–2
® T ¾¾¾
–2
® R ¾¾¾
–2
® P ¾¾¾
–2
® N
+3 +3 +3 +3 +3 +3
2nd letter: A ¾¾¾ ® D ¾¾¾ ® G ¾¾¾ ® J ¾¾¾ ® M ¾¾¾ ® P ¾¾¾ ® S

2 OVERSEAS www.allenoverseas.com
Mental Ability

The series formed by the numerals i.e. 1,2,6,21,88,445, ..... follows the pattern × 1 + 1, × 2 + 2, × 3 + 3, ×
4 + 4, × 5 + 5, .....
So, numeral in the desired term = 445 × 6 + 6 = 2676.
\ desired term is N2676S.
Hence, the answer is (1).
CONTINUOUS PATTERN SERIES
This type of questions usually consists of a series of small letters which follow a certain pattern. However, some
letters are missing from the series. These missing letters are then given in a proper sequence as one of the
alternatives. The candidate is required to choose this alternative as the answer.
Solved Examples
Direction : In each of the following letter series ,some of the letters are missing which are given in that order as
one of the alternatives below it . Choose the correct alternative.
Ex.8 aa _ ab _ _ aaa _ a
(1) aaab (2) aabb (3) abab (4) baaa
Sol. The series is aaaaba / aaaaba. Thus, the pattern aaaaba is repeated.
Hence answer is (1)

www.allenoverseas.com OVERSEAS 3
Class-VII

Directions (Q. 1 to Q.20) : In each of the following questions, a number series is given with one term missing.
Choose the correct alternative that will continue the same pattern and replace the question mark in the given
series.
1. 2, 3, 5, 7, 11, 13,?,19, 23
(1) 24 (2) 2 (3) 17 (4) 6
2. 2, 4, 7, 11, 16,?
(1) 18 (2) 20 (3) 22 (4) 25
3. 14,7,12,8,10,9, ?
(1) 7 (2) 10 (3) 11 (4) 8
4. 126,62,30,14,6, ?
(1) 1 (2) 3 (3) 2 (4) 5
5. 9, 27, 45, 63, ?, 99
(1) 108 (2) 81 (3) 72 (4) 162
6. 21, 25, 33, 49, 81, ?
(1) 97 (2) 113 (3) 129 (4) 145
7. 4, 8, 24, 96, ?, 2880
(1) 120 (2) 288 (3) 480 (4) 2976
8. 3, 4, 10, 33, 136, ?
(1) 380 (2) 650 (3) 875 (4) 685
9. 2, 6, 12, 20, 30, 42, ?
(1) 56 (2) 54 (3) 50 (4) 62
10. 75, 73, 69, 63, ?, 45, 33
(1) 132 (2) 6 (3) 55 (4) 66
11. 250, 184, 129, ?, 52, 30
(1) 313 (2) 55 (3) 85 (4)100
12. 4, 8, 12, 20, ?, 52, 84, 136
(1) 8 (2) 36 (3) 48 (4) 32
13. 6, 8, 12, 18, 26, ?, 48, 62.
(1) 44 (2) 74 (3) 36 (4) 52
14. 8, 4, 12, 6, 18, ?,27, 10, 37
(1) 9 (2) 12 (3) 18 (4) 24
15. 3, 6, 12, 21, 33, ?,66, 87
(1) 54 (2) 12 (3) 33 (4) 48
16. 4, 16, 8, 12, 12, 8, 16, ?, 20, 0
(1) 24 (2) 4 (3) 8 (4) 18
17. 250, 249, 245, 236, 220, ?
(1) 16 (2)195 (3)200 (4)180
18. 2, 5, 8, 11, 14, ?
(1) 18 (2) 17 (3) 19 (4) 16
19. 2, 5, 12, 23, 38, 57,?
(1) 69 (2) 76 (3) 80 (4) 84
20. 1, 2, 5, 12, 27, 58,?
(1) 121 (2) 136 (3) 135 (4) 174

4 OVERSEAS www.allenoverseas.com
Mental Ability

Directions (Q.21 to Q.30) : Find the missing term in each of the following:
21. D, F, I, M, ?
(1) P (2) O (3) R (4) S
22. D, I, L, Q, T, Y, B, G, ?
(1) H (2) J (3) R (4) Q
23. AZ, CX, EV, ?
(1) GS (2) GT (3) HT (4) HU
24. BF, CH, ?,HO, LT
(1) EM (2) DN (3) FJ (4) EK
25. AZ, BY, CX,?
(1) EF (2) GH (3) DE (4) DW
26. QYK, ?, ISG, EPE
(1) NWJ (2) MVI (3) NVI (4) MVJ
27. KNQ, LOR, MPS,?
(1) LOI (2) NQT (3) ZMY (4) CPR
28. CED, GIH, KML, ?
(1) OQP (2) ORQ (3) OPQ (4) SUT
29. JNR, KOS, LPT,?
(1) MQU (2) MUV (3) UZM (4) MYQ
30. BWDV, FUHT, JSLR, ?
(1) MPQQ (2) NQPP (3) HQFN (4) ABCD
Directions (Q.31 to Q.35) : In each of the following questions, a letter-number series is given with one or
more terms missing as shown by (?). Choose the missing term out of the given alternatives :
31. N5V, K7T,?, E14P, B19N
(1) H9R (2) H10Q (3) H10R (4) I10R
32. J2Z, K4X, L7V,?, N16R, O22P
(1) I11T (2) M11S (3) P12T (4) M11T
33. 2, A, 9, B, 6, C,13, D,?
(1) 9 (2) 10 (3) 12 (4) 19
34. 3F, 6G, 11I, 18L,?
(1) 21O (2) 25N (3) 25P (4) 27P
35. C4X, F9U, I16R,?
(1) K25P (2) L25P (3) L25O (4) L27P
Directions (Q.36 to Q.40 ) : In each of the following letter series, some of the letters are missing which are
given in that order as one of the alternatives below it. Choose the correct alternative.
36. b___ c a b c ___a c a ___c
(1) abb (2) bab (3) baa (4) bca
37. M__N M __ N N M M N__ M
(1) MMN (2) NNM (3) NMN (4) MNM

www.allenoverseas.com OVERSEAS 5
Class-VII

38. G H __H H G G__ G H H G__ H G H H G

(1) GGG (2) GHG (3) HHG (4) HHH


39. x__y z __ y y z x y y__
(1) xxy (2) yzz (3) yxz (4) yyx
40. p r __s __ p s q p __q s __ s q
(1) aprq (2) prrq (3) rprq (4) qrrp

Que. 1 2 3 4 5 6 7 8 9 10 11 12 13 14 15 16 17 18 19 20
Ans. 3 3 4 3 2 4 3 4 1 3 3 4 3 1 4 2 2 2 3 1
Que. 21 22 23 24 25 26 27 28 29 30 31 32 33 34 35 36 37 38 39 40
Ans. 3 2 2 4 4 2 2 1 1 2 3 4 2 4 3 1 3 2 3 4

6 OVERSEAS www.allenoverseas.com
Mental Ability

CHAPTER 2 LOGICAL VENN DIAGRAM

Venn diagram is a pictorial representation of classes representing items and their common properties. We
usually circles to draw a venn diagram. A venn diagram consists of two or more circle. Circles may or may not
have some common regions according as the respective classes have or do not have common properties amongst
them.
This section deals with questions which aim at analysing a candidate's ability to relate a certain given group of items
and illustrate it diagramatically. Here are a few different types of Venn diagrams with their implications made clear.
Suppose you are given a group of three items. Then,
1. If the items evidently belong to three different groups, the Venn
diagram representing it would be as shown alongside. A B
Ex. Doctors, Engineers, Lawyers Lawyers
Doctors
These three items bear no relationship to each other. So they are C Engineers
represented by 3 disjoint figures as shown in fig.
2. If one item belongs to the class of the second and the second
belongs to the class of third, then the representation is in the
form of three concentric circles, as shown in fig.
Hours
Ex. Seconds, Minutes, Hours C
B Minutes
Clearly, seconds are a part of minutes and minutes are a part of hours. A
Second
So, the Venn diagram would be as shown in the adjoining figure with
circle A representing Seconds, circle B representing Minutes and circle
C representing Hours.
3. If two separate items belong to the class of the third, they are
represented by two disjoint circles inside a bigger circle as
shown in fig.
Ex. Table, Chair, Furniture
Clearly, table and chair are separate items but both are items of furniture. Table A B Chair
So, they would be represented as in the adjoining figure with circle A
C Furniture
representing Table, circle B representing Chair and circle C representing
Furniture.
4. If two items belong to the class of the third such that some
items of each of these two groups are common in relationship,
then they are represented by two intersecting circles enclosed
within a bigger circle.
Ex. Males, Fathers, Brothers
Clearly, some fathers may be brothers and vice-versa. So, fathers and Fathers Brothers
A B
brothers would be represented by two intersecting circles. Also both
C Males
fathers and brothers are males. So, the diagrammatic representation
would be as shown in fig., with circle A representing Fathers, circle B
representing Brothers and circle C representing Males.

www.allenoverseas.com OVERSEAS 7
Class-VII

5. If two items are partly related to the third, and are themselves
independent of each other they are represented by three
intersecting circles in a line.

Ex. Dogs, Pets, Cats


Clearly, some dogs and some cats are pets. But all the pets are not dogs A B C
or cats. Also dogs and cats are not related to each other. So, the given
items would be represented as shown in fig. with circle A representing Dogs Pets Cats
Dogs, circle B representing Pets and circle C representing Cats.

6. If the three items are partly related to each other, they are
represented as shown in the adjoining figure. Cricket Government
fans Employees
Ex. Cricket fans, Government Employees, Educated Persons Clearly, some

Cricket fans may be government employees and some may be educated. A B


Similarly, some government employees may be Cricket fans and some C
may be educated. Also, some educated persons may be Cricket fans
Educated
and some may be government employees. So, the given items may be Persons
represented as shown in fig. with three intersecting circles denoting the
three classes.

7. If one item belongs to the class of second while third item is


entirely different from the two, then they may be represented Human Beings
by the adjoining diagram.
B
Ex. Engineers, Human Beings, Rats A C
Clearly, all engineers are human beings. This would be represented by
two concentric circles. But the class of rats is entirely different from Engineers Rats
these two. Thus, these items would be represented as shown in fig. with
circle A representing Engineers, circle B representing Human beings
and circle C representing Rats.
8. If one item belongs to the class of second and the third item is
partly related to these two, they are represented as shown
alongside. Females
Ex. Females, Mothers, Doctors B
Clearly, all mothers are females. This would be represented by two A C
concentric circles. But, some females and some mothers can be doctors.
So, the circle representing doctors would intersect each of the two
concentric circles. Thus, the diagram becomes as shown in fig. with Mothers Doctors
circle A representing Mothers, circle B representing Females and circle
C representing Doctors.

8 OVERSEAS www.allenoverseas.com
Mental Ability

9. If one item belongs to the class of second and the third item is partly
related to the second, they are represented as shown alongside. Males
Ex. Males, Fathers, Children B
Clearly, all fathers are males. This would be represented by two concentric A C
circles. But, some males are children. But, children cannot be fathers. Thus,
the diagram becomes as shown in Fig. with circle A representing Fathers, Children
Fathers
circle B representing Males and circle C representing Children.
10. If two items are partly related to each other and the third item is
entirely different from the two, they are represented as shown
Author
alongside.
Ex. Professor, Author, Children
Clearly, some professors can be authors and vice versa. This would be A B C
represented by two intersecting circles. But the class of children would be
entirely different from these two. Thus, the venn diagram would be as shown Professor Children
in fig. with circle A representing Professors, circle B representing Authors and
circle C representing Children.
Let us illustrate venn diagrams with the help of following example.

Solved examples
Ex.1 Which of the following diagram represents the best relation between Truck, Ship, Products'?

(1) (2)

(3) (4)

Sol. Some products can be transport through truck and some can through ship.

Truck
Products

Ship

Hence, the answer is (2).

www.allenoverseas.com OVERSEAS 9
Class-VII

Ex.2 Which number is in all the geometrical figures?

6
7

1 3 4 5
8
2

(1) 3 (2) 4 (3) 5 (4) 6


Sol. Clearly, we have to find a number which lies inside the triangle, the rectangle and the circle, which is 3.
Hence, the answer is (1).
Ex.3 Which of the following diagram indicates the best relation between Hockey, Football and Cricket?

(1) (2) (3) (4)

Sol. Clearly, Hockey, Football and Cricket are different sports. Hence, the answer is (2).
Ex.4 Which of the following Venn diagrams correctly illustrates the relationship among the classes: Tennis fans,
Cricket player and Students?

(1) (2) (3) (4)

Sol. Some Tennis fans are cricket players as well as students. Hence, the answer is (4).

10 OVERSEAS www.allenoverseas.com
Mental Ability

Direction (Q.1 to Q.5) : In each of these questions, three words are related in some way. The relationship
among the words in the question can best be represented by one of the four diagrams (1), (2), (3) and (4) given
below. Mark your answer accordingly.

(1) (2) (3) (4)

1. Authors, Professors, Males


2. Cabinet Minister, Home Minister, Minister
3. Professor, Researcher, Scientist
4. Rhombus, Quadrilaterals, Polygons
5. English, Latin, Greek
Direction (Q.6 to Q.11) : Each of these questions below contains three groups of things. You have to choose
from the following four diagrams, the one that depicts the correct relationship among the three groups of things
in each question.

(1) (2) (3) (4)

6. Doctors, Females, CA
7. Parents, Mother, Father
8. Languages, French, German
9. Asia, World, Universe
10. Protons, Electrons, Atoms
11. Science, Physics, Chemistry
Direction (Q.12 to Q.14) : Study the following diagram to answer these questions.

5 3
8
2 4
1
9

12. Find out the number that lies inside all the figures.
(1) 2 (2) 5 (3) 9 (4) No such number is there
13. What are the numbers that lie inside any two figures?
(1) 2, 1, 3 (2) 5, 1, 8 (3) 5, 9, 1 (4) 9, 1, 4
14. Find out the number that lies only inside the triangle.
(1) 1 (2) 2 (3) 5 (4) 9

www.allenoverseas.com OVERSEAS 11
Class-VII

Direction (Q.15 to Q.18) : In the figure given below, there are three intersecting circles each representing
certain section of people. Different regions are marked a-g. Read the statements in each of the following
questions and choose the letter of the region which correctly represents the statement.
A B
Chinese Painter
a b f
c
d e
g

C Musicians
15. Chinese who are painters but not musicians :
(1) b (2) c (3) d (4) g
16. Painters who are neither Chinese nor musicians :
(1) b (2) c (3) f (4) g
17. Chinese who are musicians but not painters :
(1) d (2) c (3) b (4) a
18. Chinese who are painters as well as musicians :
(1) a (2) b (3) c (4) d
Direction (Q.19 & Q.20) : In the following questions, answers are to be based on the diagram given below,
where the Triangle represents Doctors, the Circle represents Players and the Rectangle represents Artists.

8
7 4 3
6 5
2
1

19. Which number space in the diagram represents Doctors who are also Players and Artists?
(1) 2 (2) 3 (3) 4 (4) 5
20. Which number represents Artists who are also Players only?
(1) 4 (2) 6 (3) 7 (4) 8

Direction (Q.21 to Q.25): Find the suitable diagram among the following which represents interrelation-
ship among the following?

(1) (2) (3) (4)

21. Males, Fathers, Females


22. Soldier , Army, Engineer.
23. Males, Nephews, Cousins.
24. Doctors, Human Beings, Medicine.
25. Thieves, Pick Pockets, Black Mailers.

12 OVERSEAS www.allenoverseas.com
Mental Ability

26. Which one of the following diagrams best illustrates the three classes : Teachers, educated person, Human
being?

(1) (2) (3) (4)

27. The given three elements are related in some way. Find the diagram in which these three elements fit.
Blue, Green, Colour

(1) (2) (3) (4)

28. Study the Venn diagram carefully and answer the question given below.

Employed people
8
Backward people 11 3 6
17 5 7
Educated people
Which region represents the backward uneducated people those who are employed?
(1) 14 (2) 5 (3) 7 (4) 11
29. Which of the following Venn diagrams depicts the relationship amongst Circle, Cuboid, Triangle?

(1) (2) (3) (4)

30. Which of the following Venn diagrams depicts the relationship amongst White, food, Clothes?

(1) (2) (3) (4)

Q ue. 1 2 3 4 5 6 7 8 9 10 11 12 13 14 15 16 17 18 19 20
Ans. 3 4 3 4 2 2 4 4 1 4 4 1 2 4 1 3 1 3 4 2
Q ue. 21 22 23 24 25 26 27 28 29 30
Ans. 3 4 4 3 1 1 4 2 4 2

www.allenoverseas.com OVERSEAS 13
Class-VII

Important Notes

14 OVERSEAS www.allenoverseas.com
Mental Ability

CHAPTER 3 NON-VERBAL SERIES


The word "series" is defined as anything that follows to form a specific pattern or in continuation of a given
pattern or sequence. In this type of non-verbal test, two sets of figure state the problems. The sets are called
Problem figures and answer figures. Each problem figure changes in design from the preceding one. The
answer figure set contains 4 figures marked (1), (2), (3) and (4). You are required to choose the correct answer
figure, which would best continue the series.

Solved examples

Direction : Study the problem figures and try to establish the relationship between them. From the answer
figures marked (1), (2), (3) and (4) pick out the figure which would complete the series.

1. Problem Figure

(A) (B) (C)

(1) (2) (3) (4)

Sol. Note the direction of arrow which change alternately. The dots are also changing alternately. Hence we are
looking for a figure in which the arrow points down and the dots positioned as in figure (B). Hence, the answer
is (4).

2. Problem Figure

(A) (B) (C) (D)

Answer Figure

(1) (2) (3) (4)

Sol. Two line segments are added to A to obtain B and one line segment is added in B to obtain C. This process is
repeated again to obtain D. Hence, answer figure (4).

www.allenoverseas.com OVERSEAS 15
Class-VII

3. Problem Figure

(A) (B) (C)


Answer Figure

(1) (2) (3) (4)


Sol. The small circles are decreasing consecutively and the black dots are increasing. Hence, the answer is (3).
4. Problem Figure

+ ++ + ++

(A) (B) (C)

Answer Figure

++
++ ++ ++ ++
+ +

(1) (2) (3) (4)

Sol. Signs of plus are adding up one by one. Figure (A) has one plus signs, Figure (B) has two signs, Figure (C) has
three signs. The next figure should have four plus signs to maintain the continuity. Hence, the answer is (4).
5. Problem Figure

(A) (B) (C)


Answer Figure

(1) (2) (3) (4)


Sol. Here a decreasing trend is followed. The second figure has 7 lines. The third figure has 6 lines. To continue the
series, fourth figure should have 5 lines. Lines are disappearing one by one in the opposite side. Hence, the
answer is (4).

16 OVERSEAS www.allenoverseas.com
Mental Ability

Directions ( Q.1 to Q.15) : Study the problem figures marked A, B, C, D and E carefully and try to establish
the relationship between them. From the answer figures marked (1), (2), (3) and (4). Pick out the figure which
would complete the series.
Problem Figure Answer Figures

1.

(A) (B) (C) (D) (E) (1) (2) (3) (4)

2.

(A) (B) (C) (D) (E) (1) (2) (3) (4)

3.

(A) (B) (C) (D) (E) (1) (2) (3) (4)

4.

(A) (B) (C) (D) (E) (1) (2) (3) (4)

5.

(A) (B) (C) (D) (E) (1) (2) (3) (4)

6.

(A) (B) (C) (D) (E) (1) (2) (3) (4)

7.
T T T ?

(A) (B) (C) (D) (E) (1) (2) (3) (4)

8.

(A) (B) (C) (D) (E) (1) (2) (3) (4)

www.allenoverseas.com OVERSEAS 17
Class-VII

9.
?
(A) (B) (C) (D) (E) (1) (2) (3) (4)

10.
? ? ?? ??

(A) (B) (C) (D) (E) (1) (2) (3) (4)

11.

(A) (B) (C) (D) (E) (1) (2) (3) (4)

12.

(A) (B) (C) (D) (E) (1) (2) (3) (4)

13.

(A) (B) (C) (D) (E) (1) (2) (3) (4)

14.
?
(A) (B) (C) (D) (E) (1) (2) (3) (4)

15.

?
(A) (B) (C) (D) (E) (1) (2) (3) (4)

Que. 1 2 3 4 5 6 7 8 9 10 11 12 13 14 15
Ans. 3 2 4 1 4 3 1 3 2 2 3 2 4 2 1

18 OVERSEAS www.allenoverseas.com
Mental Ability

CHAPTER 4 CODING & DECODING


A code is a 'system of signals'. This means a coding is a method of transmitting a message between the sender
and the receiver without a third person knowing it. The coding and decoding test is set up to decipher the rule
that codes a particular word/message and break the code to decipher the message.
NUMBER/SYMBOL CODING
In these questions, either numerical code values are assigned to a word or alphabetical code letters are assigned
to the numbers. You are required to analyse the code as per the questions.
The letters and numbers would be correlated to each other in either of the following ways :
(1) Direct coding.
(2) In relation to the position of letters in English alphabet.
Solved Examples
Ex.1 If PAINT is coded as 74128 and EXCEL is coded as 93596, then how would you encode ACCEPT?
(1) 978554 (2) 545978 (3) 455987 (4) 455978
Sol. Clearly, in the given code, the alphabets are coded as follows
P A I N T E X C E L
7 4 1 2 8 9 3 5 9 6
So, in ACCEPT, A is coded as 4, C as 5, E as 9, P as 7 and T as 8. Hence, the answer is (4).
Ex.2 In a certain code, RAIN is written as 8 $%6 and MORE is written as 7 #8@. How is REMAIN written in that code?
(1) #@7 $%6 (2) #@&$%6 (3) 7@#$%6 (4) 8@7$%6

Sol. Letters R A I N M O R E

Code 8 $ % 6 7 # 8 @

Thus, the code for REMAIN is 8@7$%6. Hence, the answer is (4).
Ex.3 If PALAM could be given the code number 43, what code number can be given to SANTACRUZ?
(1) 75 (2) 85 (3) 120 (4) 123
Sol. In the given code, A=1, B=2, C=3, ............... Z=26
So, PALAM=16+1+12+1+13=43
Similarly, SANTACRUZ=19+1+14+20+1+3+18+21+26=123
Hence, the answer is (4).
Ex.4 If GO = 32, SHE = 49, then SOME will be equal to –
(1) 56 (2) 58 (3) 62 (4) 64
Sol. Counting number from opposite side
A B C D E and so on

26 25 24 23 22
The word GO can be
Coded as,
G O

20 + 12 = 32

www.allenoverseas.com OVERSEAS 19
Class-VII

and SHE as
S H E

8 + 19 +22 = 49
Similarly, SOME can be coded as

Hence, the answer is (1).


LETTER CODING
In these questions, the letter in a word are replaced by certain other letters according to a specific rule to form
its code. You are required to detect the coding pattern/rule and answer the questions accordingly.
The letters and codes would be related to each other in either of the following ways :–
(1) Direct coding.
(2) Movement of letters relative to their position in English alphabet.
(3) Reordering of letters.
Solved Examples
Ex.5 If the word EARTH be written as QPMZS in coded form, how can be HEART be written in that code?
(1) SQPZM (2) SQMPZ (3) SPQZM (4) SQPMZ
Sol. We have,

Letter E A R T H

Code Q P M Z S

So the code for HEART becomes SQPMZ.


Hence, the answer is (4).
Ex.6 In a certain code, SIKKIM is written as THLJJL. How is TRAINING written in that code?
(1) SQBHOHOH (2) UQBHOHOF (3) UQBJOHHO (4) UQBJOHOH

Sol. S I K K I M
+1 –1 +1 –1 +1 –1
T H L J J L

Clearly, the letters in the world SIKKIM are moved alternately one step forward and then one step backward to
obtain the letters of the code.

T R A I N I N G
Similarly, 'TRAINING' can be coded as +1 –1 +1 –1 +1 –1 +1 –1
U Q B H O H O F

Hence, the answer is (2).


Ex.7 In a certain code EXPLAINING is written as PXEALNIGNI, how is 'PRODUCED' written in that code?
(1) ORPUDDEC (2) ROPUDECD (3) ORPUDECD (4) DORPDECU

20 OVERSEAS www.allenoverseas.com
Mental Ability

Sol. To code the word 'EXPLAINING' the first three letter were written in reverse order, then the next two letters in
reverse order, then the next two letters and finally the last three letters in reverse order.
Similarly, 'PRODUCED' can be coded as :
EXP – LA – IN – ING PRO DU CED

PXE – AL – NI – GNI
ORP UD DEC
Hence, the answer is (1).
SUBSTITUTION
In these questions, some particular words are assigned certain substituted names. Then a question is asked that
is to be answered in the substituted code language.
Solved Examples
Ex.8 If 'sky' is called 'sea', 'sea' is called 'water', 'water' is called 'air', 'air' is called 'cloud' and 'cloud' is called 'river', then
what do we drink when thirsty?
(1) Sky (2) Air (3) Water (4) Sea
Sol. When we feel thirsty we drink water, and water is called air. Thus one would drink air.
Hence, the answer is (2).
Ex..9 If 'eraser' means 'compass' 'compass' means 'pencil', 'pencil' means 'sharpener' and 'sharpener means 'bag', then
what will a child write with?
(1) Pencil (2) Compass (3) Sharpener (4) Bag
Sol. The child use pencil to write. But 'compass' means 'pencil', so according to the question child write with compass.
Hence, the answer is (2).
DECIPHERING MESSAGE CODES
In these type of questions, some messages are given in the coded language and the code for a particular word
or message is asked. To analyse such codes, any two messages bearing a common word are picked up. The
common code-word will thus represent that word. Proceeding similarly by picking up all the possible
combinations of two, the entire message can be decoded and the codes for individual words found.
Solved examples
Ex.10 In a certain code, 'Kit Mit Fit' means 'I Am Laborious', 'Zit Rit Kit' means 'Laborious Is Dangerous' and 'Sit Fit Rit'
means 'Dangerous Extremely Painful' then in that language what is code for 'Is'?
(1) Kit (2) Zit (3) Rit (4) Data inadequate
Sol. In (i) and (ii), common word is 'Laborious' and the common code is 'Kit'.
Þ 'Kit' means 'Laborious'
Similarly, from (ii) and (iii), we can decipher' 'Rit' means 'Dangerous'
So, in (ii), the code left is 'Zit' and the word left is 'Is' Hence, the code for 'Is' is 'Zit'
Hence, the answer is (2).
Ex.11 In a certain code '786' means 'bring apple me', '958' means 'cut green apple' and '645' means 'bring green fruit'
then which one of the following is used for 'me'?
(1) 8 (2) 6 (3) 7 (4) Data inadequate
Sol. In (i) and (ii), common word is apple and the common code is 8
Þ 8 means 'apple'
Similarly, from (i) and (iii), we can decipher 6 means 'bring'.
Now, in (i) the word left is 'me' and the code left is 7
Þ code for 'me' is 7.
Hence, the answer is (3).

www.allenoverseas.com OVERSEAS 21
Class-VII

1. If 'RAM' is coded as 'QZL' in a certain language, then how will 'LAMP' be coded in that language?
(1) KZLO (2) MBNQ (3) KZLP (4) KLZN
2. If in a certain code, 'LUTE' is written as 'MUTE' and 'FATE' is written as 'GATE', then how will 'BLUE' be written
in that code?
(1) CLUE (2) GLUE (3) FLUE (4) SLUE
3. If 'ROSE' is coded as '6821'; 'CHAIR' is coded as '73456' and 'PREACH' is coded as '961473', what will be the
code for SEARCH?
(1) 246173 (2) 214763 (3) 214673 (4) 216473
4. If in a certain code '19078' is written as 'JPHSX' and '65432' as 'DBAIK; then how will '89235' be written in the
same code?
(1) JPDBI (2) XHVSK (3) HPJAK (4) XPKIB
5. If' 'DELHI' can be coded as 'CCIDD'; then how would you code 'BOMBAY'?
(1) AJMTVT (2) AMJXVS (3) MJXVSU (4) WZYZAX
6. In a certain code language the word 'METAL' is written as 'QIXEP'. How will the word 'IRON' be written in that
language?
(1) LURQ (2) JSPO (3) MVSR (4) NWTS
7. In a certain code language 'PORTRAIT' is written as 'TROPTIAR'. How would 'BIRTHDAY' be written in that
Code?
(1) TRIBYADH (2) YADHTRIB (3) IBTRDHYA (4) TRIBDHYA
8. In a certain code 'ROAM' is written as '5913' and 'DONE' is written as '4962'. How is 'MEAN' written in that
code?
(1) 5216 (2) 3126 (3) 3216 (4) 5126
9. In a certain code 'ROBE' is written as '5136' and 'BIND' is written as '3792'. How is 'RIDE' written in that code?
(1) 5276 (2) 5726 (3) 5376 (4) 5326
10. In a certain code 'GOLD' is written as '5124' and 'LIVE' is written as '2983'. How is 'VOID' written in that code?
(1) 8194 (2) 8394 (3) 8154 (4) 8793
11. In a certain code language 'RENOWNED' is written as 'DEENNORW'. How is 'INFORMATION' written in that
Code?
(1) FIINNAORRT (2) AFIIMNNOORT (3) AIIFNNOOTR (4) AIINNFOOTR
12. If the code for 'JOT' is 'OTY, then what is the code for 'HIS'?
(1) PQZ (2) LMY (3) ZQP (4) MNX
13. If the code for 'FOIL' is 'GPHK', then what is the code for 'TAME'?
(1) UBLD (2) RESM (3) RNFS (4) SFNR
14. If 'RESCUE' is coded as '372057' and 'PROBLEM' is coded as '9348176', then how will 'PROCURE' be coded?
(1) 9340537 (2) 9340357 (3) 3840537 (4) 3904537
15. In a certain language 'BANKER' is written as 'KERBAN'. How is 'REVEAL' written in the same language?
(1) REVLAE (2) EALVER (3) REVEAL (4) EALREV
16. In a certain code SPORADIC is written as QNORDJEB. How is TROUBLES written in that code?
(1) SQTNTFMC (2) TNQSRDKA (3) TNQSTFMC (4) None of these

22 OVERSEAS www.allenoverseas.com
Mental Ability

17. If the code for 'CHAP' is 'XSZK', then what is 'WVZU' the code for?
(1) LEAP (2) MOST (3) DEAF (4) COST
18. If 'OVER' is coded as 'QYIW', and 'UP' as 'WS', then STAR' is coded as ______.
(1) UWEV (2) UWDV (3) UWEW (4) UVBS
19. In a certain code 'DELHI' is written as 'CDKGH'. how will 'PATNA' be coded?
(1) OZTMZ (2) OZSMZ (3) QBUMB (4) OZTZM
20. If 'water' is called 'food', 'food' is called 'tree', 'tree' is called 'sky' and 'sky' is called 'floor', then on which of the
following fruit is grown?
(1) water (2) food (3) tree (4) sky
21. If 'pen' is called 'table','table' is called 'fan', 'fan' is called 'chair' and 'chair' is called 'roof', then on which of the
following will a person sit?
(1) fan (2) chair (3) roof (4) table
22. In a certain code language, 'pen pencil' is written as '$ £', 'eraser sharpener' is written as '@#' and ' pencil eraser'
is written as '$@'. Then, what is the code for 'pen'?
(1) # (2) £ (3) @ (4) $
23. In a certain code , '234' means 'spark and fire', '456' means 'spark is cause' and '258' means 'fire is effect'. Which
of the following numerals is used for 'cause?
(1) 3 (2) 4 (3) 5 (4) 6
24. In a certain code language, '134' means 'good and tasty', '478' means 'see good pictures' and '729' means
'pictures are faint'. Which of the following digits stands for 'see'?
(1) 9 (2) 2 (3) 1 (4) 8
25. In a certain code language, '253' means 'books are old', '546' means 'man is old' and '378', 'buy good books'.
What stands for 'are' in that code?
(1) 2 (2) 4 (3) 5 (4) 6

Que. 1 2 3 4 5 6 7 8 9 10 11 12 13 14 15 16 17 18 19 20
Ans. 1 1 3 4 2 3 1 3 2 1 2 4 1 1 4 3 3 3 2 4
Que. 21 22 23 24 25
Ans. 3 2 4 4 1

www.allenoverseas.com OVERSEAS 23
Class-VII

1. If E = 5, PEN = 35, then PAGE =?


(1) 27 (2) 28 (3) 29 (4) 36
2. If SHE = 32 and HE = 13 then, WE =?
(1) 20 (2) 25 (3) 28 (4) 30
3. If 'w' coded as 'a', 's' as 'r', 'a' as 'j', 'n' as 'z', 'e' as 'f' and 'r' as 'w' how will 'answer' be written?
(1) wnsaes (2) jzrafw (3) anrwas (4) wnraes
4. In a certain code LIBERATE is written as 56403170. TRIBAL will be written as?
(1) 734615 (2) 736415 (3) 136475 (4) 034615
5. If FINANCE is coded as GKQESIL, then how will BANK be coded?
(1) CBOL (2) CDRP (3) CCQO (4) CCPN
6. If FADE is coded as UZWV, then what does IFHG stand for in the same code?
(1) RUST (2) PORT (3) HERI (4) IHER
7. If Cloud means Rain , Rain means Tree , Tree means Axe , Axe means House , House means Mason, then from
which of the following wood can be obtained?
(1) Train (2) Rain (3) Axe (4) House
8. If the word PENCIL is coded as LICNEP, then how the word INKPOT would be coded?
(1) TOPINK (2) JOLQPU (3) HMKOPS (4) TOPKNI
9. The code for certain letters are indicated in the following words: TRADE-54321, BADGE-93271, GRADE-74321,
QUEUE-80101. What is the code for 'G'?
(1) 7 (2) 4 (3) 2 (4) 8
10. In a certain code language, (1) 'dugo hui mul zo' stands for 'work is very hard'; (2) 'hui dugo ba ki' for 'Bingo is very
smart'; (3) 'nano mul dugo' for 'Cake is hard'; and (4) 'mul ki qu' for 'smart and hard'. Which of the following words
stand for 'Bingo'?
(1) jalu (2) dugo (3) ki (4) ba
11. In a certain language 'mu mit es' means 'who is she' and 'elb mu es' means 'where is she'. What is the code for
'where' in this language?
(1) es (2) elb (3) mu (4) mit
12. In a certain code language TAP is written as SZO, then in the same code language, FREEZE is written as?
(1) EQDFYG (2) ESDFYF (3) GQFDYF (4) EQDDYD
13. In a certain code, FORGE is written as FPTJI. How is CULPRIT written in that code?
(1) CSJNPGR (2) CVMQSTU (3) CVNSVNZ (4) CXOSULW
14. If in a certain language, NEOMAN is coded as OGRQFT, which word will be coded as ZKCLUP?
(1) YJBKTO (2) XIAJSN (3) YIZHPJ (4) YIAQKJ

24 OVERSEAS www.allenoverseas.com
Mental Ability

Direction (Q.15 & Q.16) : If WORK is coded like 4-12-9-16 then, how will you code the following?
15. WOMAN
(1) 23-15-13-1-14 (2) 23-12-26-14-13 (3) 4-12-14-26-13 (4) 4-26-14-13-12
16. LUCKY
(1) 12-20-4-10-4 (2) 21-9-22-8-19 (3) 21-18-5-19-8 (4) None of these
17. If TOY = 60 and BOY = 42 then, PLAY =?
(1) 65 (2) 35 (3) 45 (4) 54
18. If D = 4 and COVER = 63, then BASIS =?
(1) 49 (2) 50 (3) 54 (4) 55
19. If Z = 52 and ACT = 48 then BAT =?
(1) 39 (2) 41 (3) 44 (4) 46
20. In a certain code language,
(i) 'il be pee' means 'roses are blue';
(ii) 'sik hee' means 'red flowers';
(iii) 'pee mit hee' means 'flowers are vegetables'.
How is 'vegetables are red flowers' written in that code?
(1) pee sik mit hee (2) sik pee hee be (3) il sik mit hee (4) cannot be determined

Que. 1 2 3 4 5 6 7 8 9 10 11 12 13 14 15 16 17 18 19 20
Ans. 3 3 2 2 3 1 2 4 1 4 2 4 3 3 3 4 4 2 4 1

www.allenoverseas.com OVERSEAS 25
Class-VII

Important Notes

26 OVERSEAS www.allenoverseas.com
Mental Ability

CHAPTER 5 MATHEMATICAL OPERATIONS


This section deals with questions on simple mathematical operations. Here, the four fundamental operations –
addition, subtraction, multiplication and division and also statements such as 'less than', 'greater than', 'equal to',
'not equal to', etc. are represented by symbols, different from the usual ones. The questions involving these
operations are set using artificial symbols. The candidate has to substitute the real signs and solve the questions
accordingly, to get the answer.
Problem solving by substitution
In this type, you are provided with substitutes for various mathematical symbols or numerals, followed by a
question involving calculation of an expression or choosing the correct/incorrect equation. The candidate is
required to put in the real signs or numerals in the given equation and then solve the questions as required.
Note : While solving a mathematical expression, proceed according to the rule BODMAS i.e. Brackets, Of,
Division, Multiplication, Addition, Subtraction.
Solved examples
Ex.1 If '+' means 'minus', ' × ' means 'divided by', ' ¸ ' means 'Plus' and '–' means 'multiplied by', then which of the
following will be the value of the expression 252 × 9 – 5 + 32 ¸ 92?
(1) 95 (2) 168 (3) 192 (4) 200
Sol. Putting the proper signs in the given expression, we get :
252 ¸ 9 × 5 – 32 + 92 = 28 × 5 – 32 + 92 = 140 – 32 + 92 = 232 – 32 = 200.
Hence, the answer is (4).
Ex.2 If L stands for +, M stands for –, N stands for × , P stands for ¸ , then 14 N 10 L 42 P 2 M 8 =?
(1) 153 (2) 216 (3) 248 (4) 251
Sol. Putting the proper signs in the given expression, we get :
14 × 10 + 42 ¸ 2 – 8 = 14 × 10 + 21 – 8 = 140 + 21 – 8 = 161 – 8 = 153.
Hence, the answer is (1).
Ex.3 If 20 – 10 means 200, 8 ¸ 4 means 12, 6 × 2 means 4 and 12 + 4 = 3 then 100 – 10 × 1000 ¸ 1000 + 100 ×
10 =?
(1) 0 (2) 20 (3) 1090 (4) 1900
Sol. Given that : 20 – 10 = 200. But, actually 20 × 10 = 200. So, – means ×.
Given that : 8 ¸ 4 = 12. But, actually 8 + 4 = 12. So, ¸ means +.
Given that : 6 × 2 = 4. But, actually 6 – 2 = 4. So, × means –.
Given that : 12 + 4 = 3. But, actually 12 ¸ 4 = 3. So, + means ¸.
Thus, in the given mathematical language, – means ×, ¸ means + , × means –. and + means ¸ . Putting the
correct signs, we have :
Given expression = 100 × 10 – 1000 + 1000 ¸ 100 – 10 = 1000 – 1000 + 10 – 10 = 0.
Hence, the answer is (1).

www.allenoverseas.com OVERSEAS 27
Class-VII

Ex.4 It being given that : > denotes +, < denotes –, + denotes ¸ , – denotes =, = denotes 'less than' and × denotes
'greater than ', find which of the following is a correct statement.

(1) 3 + 2 > 4 = 9 + 3 < 2 (2) 3 > 2 > 4 = 18 + 3 < 1

(3) 3 > 2 < 4 × 8 + 4 < 2 (4) 3 + 2 < 4 × 9 + 3 < 3

Sol. Using proper notations, we have :

11
(1) Given statement is 3 ¸ 2 + 4 < 9 ¸ 3 – 2 or < 1, which is not true.
2
(2) Given statement is 3 + 2 + 4 < 18 ¸ 3 – 1 or 9 < 5, which is not true.

(3) Given statement is 3 + 2 – 4 > 8 ¸ 4 – 2 or 1 > 0, which is true.

5
(4) Given statement is 3 ¸ 2 – 4 > 9 ¸ 3 – 3 or – > 0, which is not true.
2
Hence, the answer is (3).

28 OVERSEAS www.allenoverseas.com
Mental Ability

1. If '<' means 'minus', '>' means 'plus', '=' means 'multiplied by ' and '$' means 'divided by', then what would be the
value of 27 > 81 $ 9 < 6?
(1) 6 (2) 33 (3) 36 (4) None of these
2. If × means –, + means ¸ , – means × and ¸ means +, then 15 – 2 ¸ 900 + 90 × 100 =?
(1) 190 (2) 180 (3) 90 (4) None of these
3. If '+' means 'divided by', '–' means 'add', '×' means 'minus' and '/' means 'multiplied by', what will be the value of
the following expression?
[{(17 × 12) – (4/2)} + (23 – 6)]/0
(1) Infinite (2) 0 (3) 118 (4) 219
4. If '–' stands for 'division', '+' for 'multiplication', ' ¸ ' for 'subtraction' and '×' for 'addition', then which one of the
following equations is correct?
(1) 4 × 5 + 9 – 3 ¸ 4 = 15 (2) 4 × 5 × 9 + 3 ¸ 4 = 11
(3) 4 – 5 ¸ 9 × 3 – 4 = 17 (4) 4 ¸ 5 + 9 – 3 + 4 = 18
5. If '+' stands for 'division', ' ¸ ', stands for 'multiplication', '×' stands for 'subtraction' and '–' stands for 'addition', which
one of the following is correct?
(1) 18 ¸ 6 × 7 + 5 – 2 = 22 (2) 18 × 6 + 7 ¸ 5 – 2 = 16
(3) 18 ¸ 6 – 7 + 5 × 2 = 20 (4) 18 + 6 ¸ 7 × 5 – 2 = 18
6. If L denotes ¸ , M denotes ×, P denotes + and Q denotes –, then which of the following statements is true?
3 173
(1) 32 P 8 L 16 Q 4 = – (2) 6 M 18 Q 26 L 13 P 7 =
2 13
38
(3) 11 M 34 L 17 Q 8 L 3 = (4) 9 P 9 L 9 Q 9 M 9 = – 71
3
7. If '×' stands for 'addition', '<' for 'subtraction', '+' for 'division', '>' for 'multiplication','–' for 'equal to' , ' ¸ ' for 'greater
than' and '=' for 'less than', then state which of the following is true?
(1) 3 × 4 > 2 – 9 + 3 < 3 (2) 5 × 3 < 7 ¸ 8 + 4 × 1
(3) 5 > 2 + 2 = 10 < 4 × 8 (4) 3 × 2 < 4 ¸ 16 > 2 + 4
Directions (Q.8 & Q.9) : In each of the following questions, some symbols are represented by letters as
shown below.

+ – × –·· = > <


B G E C D A F

Now, identify the correct expression in each case.


8. (1) 18 C 3 D 6 B 8 C 4 G 12 (2) 18 A 3 E 6 B 8 G 4 B 12
(3) 18 C 3 G 6 B 8 B 4 D 12 (4) 18 F 3 B 6 E 8 G 4 E 12
9. (1) 15 B 5 G 8 B 4 G 6 F 3 (2) 15 C 15 B 8 F 4 B 6 C 3

(3) 15 A 5 E 8 C 4 B 6 E 3 (4) 15 C 5 F 8 C 4 B 6 C 3

www.allenoverseas.com OVERSEAS 29
Class-VII

Directions (Q.10 to Q.14) : In each of the following questions, different alphabets stand for various symbols
as indicated below :
Addition : O Subtraction : M Multiplication : A
Division : Q Equal to : X Greater than : Y
Less than : Z
Out of the four alternatives given in these questions, only one is correct according to the above letter symbols.
Identify the correct answer.
10. (1) 2 Z 2 A 4 O 1 A 4 M 8 (2) 8 Y 2 A 3 A 4 Q 2 A 4
(3) 10 X 2 O 2 A 4 O 1 M 2 (4) 12 X 4 O 2 Q 1 A 4 A 2
11. (1) 1 O 1 Q 1 M 1 Y 3 Q 1 (2) 2 Q 1 O 10 A 1 Z 6 A 4
(3) 3 O 2 O 10 Q 2 X 10 A 2 (4) 5 Q 5 A 5 O 5 Y 5 A 2
12. (1) 3 O 2 X 2 Q 1 A 3 O 1 (2) 6 M 2 Y 10 Q 2 A 3 O 1
(3) 10 A 2 Z 2 Q 2 A 10 Q 2 (4) 10 A 2 Y 2 Q 1 A 10 Q 2
13. (1) 32 X 8 Q 2 A 3 Q 1 A 2 (2) 14 X 2 A 4 A 2 M 2 Q 1
(3) 2 Y 1 A 1 Q 1 O 1 A 1 (4) 16 Y 8 A 3 O 1 A 2 M 2
14. (1) 8 Q 4 A 1 M 2 X 16 M 16 (2) 8 O 2 A 12 Q 10 X 18 Q 9
(3) 6 Q 2 O 1 O 1 X 16 A 1 (4) 2 O 3 M 4 Q 2 Y 1 A 2

15. If ¸ means × , × means +, + means – and – means ¸ , find the value of 16 × 3 + 5 – 2 ¸ 4.


(1) 9 (2) 10 (3) 19 (4) None of these

16. If + means ¸ , ¸ means – , – means × , × means +, then 12 + 6 ¸ 3 – 2 × 8 =?

(1) –2 (2) 2 (3) 4 (4) 8

17. If P denotes ¸ , Q denotes × , R denotes + and S denotes – , then what is the value of 18 Q 12 P 4 R 5 S 6?
(1) 53 (2) 59 (3) 63 (4) 65

18. If P means 'division', T means 'addition', M means 'subtraction' and D means 'multiplication', then what will be the
value of the expression 12 M 12 D 28 P 7 T 15?
(1) – 30 (2) – 15 (3) 15 (4) None of these
19. If P means × , R means +, T means ¸ and S means –, then 18 T 3 P 9 S 8 R 6 =?
1 2
(1) – 1 (2) (3) 46 (4) None of these
3 3
20. If P denotes 'multiplied by', T denotes 'subtracted from', M denotes 'added to' and B denotes 'divided by', then
28 B 7 P 8 T 6 M 4 =?
3
(1) – (2) 30 (3) 32 (4) 34
2
21. If P = 0, Q = 1, R =22, S = 35, and T = 57, then, what is R + S × P =?

(1) R (2) P (3) Q (4) S

30 OVERSEAS www.allenoverseas.com
Mental Ability

22. If P = 1, Q = 3, R = 5, and S = 9 then P + Q + R =?

(1) Q (2) P (3) S (4) R

Directions (Q.23 to.Q.24) : If `+' is `× ', `–' is `+', `× ' is `÷' and '÷' is `–', then answer the following questions
based on this information.
23. 15 × 5 ÷ 3 + 1 – 1 =?
(1) 1 (2) 2 (3) 3 (4) None of these
24. 9 – 3 + 2 ÷ 16 × 2 =?
(1) 7 (2) 5 (3) 9 (4) 6

25. If A means 'plus', B means 'minus', C means 'divided by' and D means 'multiplied by' then 18 A 12 C 6 D 2 B 5
equals_____.

(1) 17 (2) 25 (3) 27 (4) 45

26. If P means ' ÷ ', R means ' +' , T means '–' and W means '× ' , then what is the value of given statement?

24R16P8W6T9

1
(1) 15 (2) 21 (3) 147 (4) 27
3

27. If 'P' means '+' , 'Q' means ' × ' , 'R' means '÷ ' and 'S' means ' – ' , then 81 R 3 S 15 P 5 Q 0 is _______

(1) Equal to 0 (2) Less than –1 (3) Greater than 2 (4) Lessen than 0

28. If 'M' denotes ' × ', 'R' denotes '–' 'K' denotes '+' and ' B' denotes ' ÷ ' , then 24B4M8K6R4 = ?

4 1
(1) –3 (2) 50 (3) –3 (4) 52
7 4

29. Find the correct alternative to make the equation correct:

(13 ? 9) ? (14 ? 11) ? 13 = 3

(1) +, × , +, ÷ (2) –, × , –, ÷ (3) × , ÷ , –, ÷ (4) × , × , –, ÷sw

30. If ® stands for addition, ¬ stands for Subtraction ,­ stands for Division,¯ stands for Multiplication, and
stands for Equal to, then which of the following options is correct?

(1) 7¬ 43 ­ 6 ¯1 4 (2) 3 ¯ 6 ­ 2 ® 3 6

(3) 5 ®7¬ 3 ­ 2 4 (4) 2 ¯ 5 ¬ 6 ® 2 6

www.allenoverseas.com OVERSEAS 31
Class-VII

Directions (Q.31 to Q.35) : If `+' is `– ', `–' is `× ', `× ' is `÷' and '÷' is `+', then answer the following questions
based on this information.

31. 9 ÷ 5 + 4 –3 × 2 =?

(1) 2 (2) 9 (3) 8 (4) None of these

32. 7 + 6 × 3 – 8 ÷ 20 =?

(1) 3 (2) 7 (3) 2 (4) 11

33. 3 × 2 + 4 – 2 ÷ 9 =?

(1) 1 (2) 2 (3) 3 (4) None of these

34. 6 – 9 + 8 × 4 ÷ 20 =?

(1) 27 (2) 72 (3) 70 (4) None of these

35. 5 × 4 – 8 ÷ 3 + 1 =?

(1) 0 (2) 12 (3) 14 (4) None of these

Que. 1 2 3 4 5 6 7 8 9 10 11 12 13 14 15 16 17 18 19 20
Ans. 4 4 2 1 4 4 3 3 4 1 2 4 2 1 1 3 1 4 4 2
Que. 21 22 23 24 25 26 27 28 29 30 31 32 33 34 35
Ans. 1 3 1 1 1 4 3 2 3 4 3 4 4 2 2

32 OVERSEAS www.allenoverseas.com
Mental Ability

CHAPTER 6 ANALYTICAL REASONING

The chapter on Analytical Reasoning involves the problems relating to the counting of geometrical figures in
a given complex figure. The systematic method for determining the number of any particular type of figure
by the analysis of the complex figure would be clear from the examples that follow.
Solved Examples
Ex.1 What is the number of straight lines in the following figure?

(1) 10 (2) 12 (3) 13 (4) 17


Sol. We shall label the figure as shown below:
A B C

H D
I J

G F E
Clearly, in this figure : There are 3 vertical lines namely AG, BF and CE. There are 3 horizontal lines namely AC,
HD and GE. There are 6 slanting lines namely AD, AE, GC, GD, CD and ED. Thus, there are 3 + 3 + 6 = 12
straight lines in all. Hence, the answer is (2).
Ex.2 How many triangles are there in the following figure?

(1) 6 (2) 10 (3) 11 (4) 12


Sol. The figure may be labelled as shown below
B

F
E

A C
D
The simplest triangles are ABE, BEF, EFC, CDE and AED i.e. 5 in number. The triangles composed of two
components each are ABF, BCE, ACE and ABD i.e. 4 in number. The triangles composed of three components
each are AFC and BCD i.e. 2 in number. There is only one triangle ABC composed of five components.
Thus, there are 5 + 4 + 2 + 1 = 12 triangles in the figure. Hence, the answer is (4).

www.allenoverseas.com OVERSEAS 33
Class-VII

Ex.3 Count the number of squares in the following figure:

(1) 18 (2) 14 (3) 10 (4) 9

Sol. We shall label the figure as shown below:


A E F B

L G
M N

K P O H

D J I C

The simplest squares are AEML, EFNM, FBGN, NGHO, MNOP, LMPK, KPJD, POIJ and OHCI i.e. 9 in
number.

The squares composed of four components each are AFOK, EBHP, LNID and MGCJ i.e. 4 in number.

There is only one square i.e. ABCD composed of nine components.

Thus, there are 9 + 4 + 1 = 14 squares in the figure.

Hence, the answer is (2).

34 OVERSEAS www.allenoverseas.com
Mental Ability

Direction (Q1 to Q.9) : Find the minimum number of straight lines used in forming the given figure in each of
the following question.

1.

(1) 11 (2) 13 (3) 15 (4) 21

2.

(1) 5 (2) 6 (3) 8 (4) 10

3.

(1) 14 (2) 16 (3) 18 (4) 20

4.

(1) 11 (2) 12 (3) 13 (4) 14

5.

(1) 9 (2) 11 (3) 12 (4) 16

6.

(1) 10 (2) 16 (3) 21 (4) 23

7.

(1) 10 (2) 12 (3) 14 (4) 16

www.allenoverseas.com OVERSEAS 35
Class-VII

8.

(1) 12 (2) 11 (3) 10 (4) 9

9.

(1) 19 (2) 18 (3) 17 (4) 16


10. How many circles are there in the adjoining figure?

(1) 13 (2) 16 (3) 15 (4) 10


Direction (Q.11 to Q.17): Find the number of triangles in each of the given figures.

11.

(1) 12 (2) 18 (3) 22 (4) 26

12.

(1) 11 (2) 13 (3) 15 (4) 17

13.

(1) 12 (2) 13 (3) 14 (4) 15

14.

(1) 18 (2) 20 (3) 28 (4) 34

36 OVERSEAS www.allenoverseas.com
Mental Ability

15.

(1) 12 (2) 13 (3) 14 (4) None of these

16.

(1) 15 (2) 16 (3) 17 (4) 18

17.

(1) 18 (2) 24 (3) 21 (4) 20


Direction (Q.18 to Q.20) : Count the number of squares in the given figure.

18.

(1) 16 (2) 30 (3) 28 (4) 20

19.

(1) 8 (2) 9 (3) 10 (4) 11

20.

(1) 10 (2) 11 (3) 12 (4) 13


21. In the adjoining figure, if the centres of all the circles are joined by horizontal and vertical lines, then find the
number of squares that can be formed.

(1) 6 (2) 7 (3) 8 (4) 1


22. How many rectangles are there in the adjoining figure?

(1) 10 (2) 9 (3) 8 (4) 7

www.allenoverseas.com OVERSEAS 37
Class-VII

23. Count the number of triangles and squares in the given figure..

(1) 26 triangles, 5 squares (2) 28 triangles, 5 squares


(3) 26 triangles, 6 squares (4) 28 triangles, 6 squares
24. Count the number of triangles and squares in the given figure.

(1) 28 triangles, 3 squares (2) 24 triangles, 5 squares


(3) 28 triangles, 5 squares (4) 24 triangles, 3 squares
25. What is the number of straight lines and the number of triangles in the given figure.

(1) 10 straight lines and 34 triangles (2) 9 straight lines and 34 triangles
(3) 9 straight lines and 36 triangles (4) 10 straight lines and 36 triangles

Que. 1 2 3 4 5 6 7 8 9 10 11 12 13 14 15 16 17 18 19 20
Ans. 2 1 1 2 3 2 3 4 3 1 2 3 4 3 1 3 1 2 3 1
Que. 21 22 23 24 25
Ans. 3 2 3 3 3

38 OVERSEAS www.allenoverseas.com
Mental Ability

CHAPTER 7 ANALOGY
'Analogy' means 'correspondence'.
In questions based on analogy, a particular relationship is given and another similar relationship has to be
identified from the alternatives provided. Analogy tests are, therefore, meant to test a candidate's overall
knowledge, power of reasoning and ability to think concisely and accurately. Below are given some common
relationships which will help you to detect most analogies better.
COMMON RELATIONSHIPS
1. Country and capital :
Ex. Afghanistan : Kabul
Kabul is the capital of Afghanistan.
2. State and capital :
Ex. Maharashtra : Mumbai
Mumbai is the capital of Maharashtra.
3. Country and currency :
Ex. India : Rupee
Rupee is the currency of India.
4. Instrument and measurement :
Ex. Barometer : Pressure
Barometer is an instrument used to measure pressure.
5. Quantity and units :
Ex. Length : Metre
Metre is the unit of measuring length.
6. Individual and group :
Ex. Sailors : Crew
A group of sailors is called a crew.
7. Animal and young one :
Ex. Cow : Calf
Calf is the young one of cow.
8. Male and female :
Ex. Horse : Mare
Mare is the female horse.
9. Animal and movement :
Ex. Duck : Waddle
Waddling is the name given to the movement of the duck.
10. Animal/thing and sound :
Ex. Lion : Roar
Roar is the sound produced by a lion.
11. Individual/thing and class :
Ex. Lizard : Reptile
Lizard belongs to the class of Reptiles.
12. Individual and dwelling place :
Ex. Dog : Kennel
A dog lives in a kennel.

www.allenoverseas.com OVERSEAS 39
Class-VII

13. Animals/things and keeping place :


Ex. Car : Garage
A car is kept in a garage.
14. Games and place of playing :
Ex. Badminton : Court
Badminton is played on a court.
15. Worker and tool :
Ex. Blacksmith : Anvil
Anvil is the tool used by a blacksmith.
16. Tool and action :
Ex. Needle : Sew
A needle is used for sewing.
17. Worker and working place :
Ex. Chef : Kitchen
A chef works in a kitchen.
18. Worker and product :
Ex. Mason : Wall
A mason builds a wall.
19. Product and raw material :
Ex. Prism : Glass
Prism is made of glass.
20. Part and whole relationship :
Ex. Pen : Nib
Nib is a part of a pen.
21. Pair relationship :
Ex. Shoes : Socks
Shoes and socks go together.
22. Study and topic :
Ex. Ornithology : Birds
Ornithology is the study of birds.
23. Word and intensity :
Ex. Anger : Rage
Rage is of higher intensity than Anger.
24. Word and synonym :
Ex. Abode : Dwelling
Abode means almost the same as Dwelling. Thus, Dwelling is the synonym of Abode.
25. Word and antonym :
Ex. Attack : Defend
Defend means the opposite of Attack. Thus, Defend is the antonym of Attack.
COMPLETING THE ANALOGOUS PAIR
In this type of questions, two words are given. These words are related to each other in some way. Another word
is also given. You are required to find out the relationship between the first two words and choose the word from
the given alternatives, which bears the same relationship to the third word, as the first two bear.

40 OVERSEAS www.allenoverseas.com
Mental Ability

Solved Examples
Ex.1 Botany : Plants :: Entomology :?
(1) Snakes (2) Insects (3) Birds (4) Germs
Sol. Botany is the branch of science which deals with the study of plants. Similarly, Entomology is the branch of
science which deals with the study of insects. Hence, the answer is (2).
CHOOSING THE ANALOGOUS PAIR
In the type of questions, a pair of words is given, followed by four pairs of words as alternatives. You are required
to choose the pair in which the words bear the same relationship to each other as the words of the given pair
bear.
Solved Examples
Ex.2 Chimney : Smoke
(1) Tea : Kettle (2) Clay : Ceramic (3) House : Roof (4) Gun : Bullet
Sol. Smoke comes out of chimney and bullet comes out of gun. Hence, the answer is (4).

NUMBER ANALOGY
This section deals with four types of questions:

1. Choosing a number related to a given number in the same manner as the two numbers of another given
pair are related to each other.
2. Choosing a similarly related pair as the given number pair on the basis of the relation between the numbers
in each pair.

3. Choosing a number similar to a group of numbers on the basis of certain common properties that they
possess.

4. Choosing a number set similar to a given number set.

Solved Examples
Ex.3. 4 : 18 :: 8 :?
(1) 62 (2) 59 (3) 48 (4) 66
Sol. Clearly, 42 + 2 = 18, Now, 82 + 2 = 66. So, if the first number is x, the second number is x2 + 2.
Thus, the relationship is x : x2 + 2.
Hence, the answer is (4).
Ex.4. 13 : 17
(1) 2 : 3 (2) 27 : 29 (3) 7 : 10 (4) 9 : 11
Sol. Clearly, the relationship is consecutive prime numbers. Hence, the answer is (1).

ALPHABET ANALOGY
In this type of questions, two groups of letters related to each other in some way, are given. You are required to
find out this relationship and then choose either a letter-group which is related in the same way to a third group
provided in the question or a pair consisting of similarly related letter-groups.

www.allenoverseas.com OVERSEAS 41
Class-VII

Solved Examples

Ex.5 Find the matching pair.

PASS : QBTT :: FAIL :?

(1) GJBM (2) GBJM (3) MBJG (4) MJBG

+1 +1
Sol. P ¾¾¾
® Q F ¾¾¾
® G

+1 +1
A ¾¾¾
® B Similarly A ¾¾¾ ® B

+1 +1
S ¾¾¾ ® T I ¾¾¾ ® J

+1 +1
S ¾¾¾ ® T L ¾¾¾ ® M

Therefore, the matching pair is (2).

Hence, the answer is (2).

42 OVERSEAS www.allenoverseas.com
Mental Ability

Direction (Q.1 to Q.4) : In each of the following questions, there is a certain relationship between two given
words on one side of : : and one word is given on another side of : : while another word is to be found from the
given alternatives, having the same relation with this word as the words of the given pair bear. Choose the
correct alternative.
1. Museum : Curator :: Prison :?
(1) Wonder (2) Monitor (3) Manager (4) Jailor
2. Wax : Grease :: Milk :?
(1) Curd (2) Protein (3) Shake (4) Drink
3. Saudi Arabia : Riyal :: Japan :?
(1) Dollar (2) Yen (3) Pound (4) Mork
4. Lung : Man :: Gill :?
(1) Hen (2) Peacock (3) Women (4) Fish
Direction (Q.5 & Q.6) : Find the matching pair.
5. Dusk : Night
(1) Morning : Evening (2) Child : Adult
(3) Walk : Run (4) Day : Light
6. Teeth : Chew
(1) Mind : Think (2) Sweater : Heat (3) Food : Taste (4) Eyes : Flickers
Direction (Q.7 & Q.8) : Each of the following questions consists of a pair of number that have a certain
relationship to each other, followed by four other pairs of numbers that have a certain relationship to each other,
followed by four other pairs of numbers given as alterantives. Select the pair in which the numbers are similarly
related as in the given pair.
7. 27 : 9
(1) 64 : 8 (2) 125 : 5 (3) 135 : 5 (4) 729 : 81
8. 7 : 24
(1) 30 : 100 (2) 23 : 72 (3) 19 : 58 (4) 11 : 43
Direction (Q.9 to Q.12) : In each of the following questions, there is a certain relationship between two given
numbers on one side of : and one number is given on another side of : : while another number is to be found
from the given alternatives, having the same relationship with this number as the numbers of the given pair
bear. Choose the best alternative.
9. 14 : 9 : : 26 :?
(1) 12 (2) 13 (3) 15 (4) 31
10. 49 : 81 : : 100 :?
(1) 64 (2) 144 (3) 169 (4) None of these
11. 7 : 56 : : 9 :?
(1) 63 (2) 81 (3) 90 (4) 99
12. 8 : 81 : : 64 :?
(1) 125 (2) 137 (3) 525 (4) 625

www.allenoverseas.com OVERSEAS 43
Class-VII

Direction (Q.13 to Q.19) : In each of the following questions, there is some relationship between the two
terms to the left of : : and the same relationship holds between the two terms to its right. Also, in each question,
one term either to the right of : : or to the left of it is missing part. This term is given as one of the alternatives
given below each question. Find out this term.
13. MAD : JXA : : RUN :?
(1) ORK (2) OSQ (3) PRJ (4) UXQ
14. QDXM : SFYN : : UIOZ :?
(1) WKPA (2) QNLA (3) LPWA (4) PAQM
15. FILM : ADGH : MILK :?
(1) DGFH (2) HDGF (3) HFDG (4) HDFG
16. kcaC : Cack : : XgmF :?
(1) EmgF (2) EgmX (3) FmgX (4) GmeF
17. CLOSE : DNRWJ : : OPEN :?
(1) PRHR (2) PRJQ (3) RPJB (4) RZWR
18. EIGHTY : GIEYTH : : OUTPUT :?
(1) TUOTUP (2) TUOUTP (3) UTOPTU (4) UOTUPT
19. computer : fqprxvht : : language :?
(1) oxpixdig (2) ocqicyig (3) ocqixcjg (4) ocqixcig
20. EGIK is related to WUSQ in the same way as DFHJ is related to ........?
(1) BDFH (2) ECGI (3) SQOM (4) XVTR
Direction (Q.21 to Q.25) : In each of the following questions, there is some relationship between the two
terms to the left of : : and the same relationship holds between the two terms to its right. Also, in each question,
one term either to the right of : : or to the left of it is missing part. This term is given as one of the alternatives
given below each question. Find out this term.
21. 6 : 18 :: 4 :?
(1) 12 (2) 6 (3) 8 (4) 16
22. 21 : 3 :: 700 :?
(1) 23 (2) 102 (3) 100 (4) 93
23. 42 : 21 :: 28 :?
(1) 31 (2) 14 (3) 33 (4) 56
24. 7500 : 7400 :: 4600 :?
(1) 2367 (2) 2451 (3) 2531 (4) 4500
25. 12 : 60 :: 21 :?
(1) 165 (2) 205 (3) 105 (4) 118
Direction (Q.26 to Q.29) : Find the matching pair :
26. JK25 : LM50
(1) 25JK : 75LM (2) AB35 : DE70 (3) XY19 : 38YX (4) PQ12 : RS24
27. AB12 : BC24
(1) EF10 : GH30 (2) XY36:YZ72 (3) LM15 : ML30 (4) RS13 : SR39
28. 3GH : 9HG
(1) 5JK : 10KJ (2) 9LM : 36ML (3) 6PQ : 18QP (4) 4RS : 12TU

44 OVERSEAS www.allenoverseas.com
Mental Ability

XY YZ
29. :
9 7

AB BC LM NO HI JK PQ RS
(1) : (2) : (3) : (4) :
6 4 3 6 4 8 10 6

Direction (Q.30 to Q.32) : In each of the following questions, there is some relationship between the two
terms to the left of : : and the same relationship holds between the two terms to its right. Also, in each question,
one term either to the right of : : or to the left of it is missing part. This term is given as one of the alternatives
given below each question. Find out this term.
30. Fear : Threat : : Anger :?
(1) Compulsion (2) Panic (3) Provocation (4) Force
31. 9 : 8 : : 16 :?
(1) 27 (2) 18 (3) 17 (4) 14
32. 18 : 48 : : 180 :?
(1) 294 (2) 230 (3) 392 (4) 292
33. Find the matching pair.
Blister : Skin
(1) Sore : Toe (2) Sty : Eye (3) Ball : Pitcher (4) Wound : Arm
34. Find a number similar to the numbers given.
134 , 642 ,358
(1) 372 (2) 460 (3) 522 (4) 927
35. Which set of numbers in the options is like the set given below?
121, 81, 9
(1) 95,24,5 (2) 64,9,4 (3) 168,15,3 (4) 10,6,5

Que. 1 2 3 4 5 6 7 8 9 10 11 12 13 14 15 16 17 18 19 20
Ans. 4 1 2 4 2 1 4 2 3 2 3 4 1 1 2 3 1 1 3 4
Que. 21 22 23 24 25 26 27 28 29 30 31 32 33 34 35
Ans. 3 3 2 4 3 4 2 3 1 3 1 1 2 4 2

www.allenoverseas.com OVERSEAS 45
Class-VII

Important Notes

46 OVERSEAS www.allenoverseas.com
Mental Ability

CHAPTER 8 CLASSIFICATION

Classification means to assort the items of a given group on the basis of certain common quality they possess and
then sport the stranger out.

These test judge your ability to observe differences and similarities among items. You must determine the
similarity of characteristics in the given terms and then identify the one which does not have same characteristics.
These characteristics can be based on relationship, small and capital letter relationship, vowel and consonant
relationship, repetitions and skipping pattern relationship, letter formation relationship.

1. Choosing the odd word

Solved examples

Ex.1 Find the odd one out :

(1) Mango (2) Pear (3) Orange (4) Lemon

Sol. Remaining three are fruit except Lemon. Hence, the answer is (4).

Ex.2 (1) Bake (2) Peel (3) Fry (4) Boil

Sol. Here, all except Peel are different forms of cooking. Hence, the answer is (2).

2. Choosing the odd pair of words

Ex.3 (1) Volume : Litre (2) Pressure : Barometer

(3) Length : Metre (4) Resistance : Ohm

Sol. In all other pairs, second is the unit to measure the first. On the other hand, barometer is an instrument. Hence,
the answer is (2).

3. Choosing the odd numeral

Ex.4 (1) 48 (2) 50 (3) 82 (4) 170

Sol. Each of the numbers except 48, is one more than the square of a certain number. Hence, the answer is (1).

4. Choosing the odd numeral pair/group

Ex.5 (1) 71,7, 3, 17 (2) 67, 71, 3, 5 (3) 41,5,3,47 (4) 37, 14, 19, 7

Sol. All other groups except (4) consist of prime numbers only, while (4) consists of one composite number i.e. 14.
Hence, the answer is (4).

www.allenoverseas.com OVERSEAS 47
Class-VII

5. Choosing the odd letter group

Ex.6 (1) CZHK (2) MLAG (3) XUBU (4) SENO

Sol. Option (3) is the only group in which one letter has been repeated. Hence, the answer is (3).

Ex.7 (1) BDGK (2) JLOS (3) NPSW (4) MORU

Sol. In all other groups, there is a gap of 1 letter between first and second letters, 2 letters between second and third
letters, and 3 letters between third and fourth letters. Hence, the answer is (4).

48 OVERSEAS www.allenoverseas.com
Mental Ability

Direction (Q.1 to Q.35) :You are given a group of certain items, out of which, all except one are similar to
one another in some manner. You are required to choose this one item which does not fit into the given group.
1. (1) Kiwi (2) Eagle (3) Ostrich (4) Penguin
2. (1) Arrow (2) Axe (3) Dagger (4) Sword
3. (1) Sun (2) Moon (3) Earth (4) Mountain
4. (1) Sea (2) River (3) Swimming pool (4) Lake
5. (1) Bear (2) Cow (3) Fox (4) Lion
6. (1) Dinar (2) Dollar (3) Yen (4) Vatican
7. (1) Yellow (2) Blue (3) Brown (4) Indigo
8. (1) Paper (2) Pencil (3) Sharpner (4) Encyclopedia
9. (1) Badminton (2) Tennis (3) Cricket (4) Diving
10. (1) Student (2) Lawyer (3) Doctor (4) Engineer
11. (1) Fan (2) Cooler (3) Fridge (4) Air Conditioner
12. (1) Arc (2) Diagonal (3) Tangent (4) Radius
13. (1) Moscow (2) London (3) Tokyo (4) Mumbai
14. (1) Hydrometer (2) Barometer (3) Thermometer (4) Diameter
15. (1) Up (2) Down (3) Below (4) Small
16. (1) Write (2) Read (3) Knowledge (4) Learn
17. (1) Run (2) Walk (3) Think (4) Jump
18. (1) Bake (2) Peel (3) Fry (4) Roast
19. (1) Calendar (2) Year (3) Date (4) Month
20.
AC CE BD EH
(1) (2) (3) (4)
21.
AA BD CI FS
(1) (2) (3) (4)
22.
YA XC UF TG
(1) (2) (3) (4)
23.

24.

25.

www.allenoverseas.com OVERSEAS 49
Class-VII

26.

27.
761 913 615 775
(1) (2) (3) (4)

28.

29.
49 36 100 16
(1) (2) (3) (4)

30.
600 640 700 800
(1) (2) (3) (4)

31. (1) Malaria (2)Plague (3) Dengue (4) Tetanus


32. (1) 22 (2) 33 (3) 11 (4) 44
33. (1) Fan : Blades (2) Lamp: Bulb (3) Clock : Alarm (4) Bicycle : Pedal
34. (1) O E 31 (2) X F 30 (3) J B 12 (4) P H 24
35. (1) 21 : 6 (2) 28 : 4 (3) 42 : 12 (4) 84 : 24

Que. 1 2 3 4 5 6 7 8 9 10 11 12 13 14 15
Ans . 2 1 4 3 2 4 3 4 4 1 3 1 4 4 4
Que. 16 17 18 19 20 21 22 23 24 25 26 27 28 29 30
Ans . 3 3 2 1 4 4 1 4 4 1 4 1 4 1 2
Que. 31 32 33 34 35
Ans . 4 3 3 1 2

50 OVERSEAS www.allenoverseas.com
Mental Ability

CHAPTER 9 NON-VERBAL ANALOGY


Objective
To spot a connection between two objects.
The question types that make up this set are called analogies. In both verbal and non-verbal reasoning, in
analogy questions, you will usually be given one pair of images that are connected in a particular way and the
first image of a second pair. You have to find the correct image to complete the second pair in the same way as
the first pair. As with the odd one out, analogies can be based on a variety of different connections.
Solved examples
Ex.1 Find the matching pair.

?
(1) (2) (3) (4)

Sol. Look carefully at first pair and consider their relationship to the second pair.
Shape : Both are of the same shape, and both contain a circle and a square.
Position : The position of the square ( ) has changed from top of the left side to the bottom of the right side.
The position of the circle ( ) has changed from bottom of the left side to the top of the right side.
Angle : No change in any of the angles.
Number : No change in any number.
Shading : No change in the shading.
Size : No change in size.
Now, you can predict the second shape in the second pair.
Hence, the answer is (2).
Ex.2 Which shape completes the second pair in the same way as the first pair?

?
(1) (2) (3) (4)

Sol. Compare the first pair of shapes. What do you notice?


The second shape is made up of the same lines and circle as the first one but there is something different.
The short horizontal lines are on the opposite sides of the vertical line in the second shape. Can you think of
anything that might cause this effect?
Imagine that a mirror has been placed on the dotted line shown below. How would the mirror image of the
shape look like?

www.allenoverseas.com OVERSEAS 51
Class-VII

Original Reflection

The link in the first pair is to reflection. The second shape is the mirror image or reflection of the first shape. In
this example, the first shape of the second pair has several element. So, it may appear more difficult to work
over.
Original Reflection

?
This rule of reflection can now be applied to the second pair.
For complex shapes like this, try to look at each section individually and find its reflection in a mirror. This will
help you to build a full picture of the answer.
Here, you might start by thinking about the vertical arrows and the circle in the middle. How would they look like
in a mirror?
The reflection of these shapes would look the same.
Original Reflection

Next, you could look at the diagonal lines. How would these look like?
These would also appear the same in mirror.
Original Reflection

You could then think about the horizontal arrows one at a time. Would the reflection of the arrow look different
from the original? Yes, the arrow would be facing the opposite direction in the reflection.
Original Reflection

Lastly, think about the white - headed arrow. Would this change?
Yes, this arrowhead would also change its direction.
Now that you have thought about how each part of the symbol would look in a mirror, you need to put all of the
elements together to find the answer.

Hence, the answer is (3).

52 OVERSEAS www.allenoverseas.com
Mental Ability

Ex.3 Find the matching pair.

?
(1) (2) (3) (4)

Sol. In the first pair the pentagon rotates 90° clockwise and a square having four sides covers it. Similarly, in the
second pair, the square (having four sides) rotates 90° clockwise and will be covered by a triangle (having three
sides).
Hence, the answer is (1).
Ex.4 Which shape or pattern completes the second pair in the same way as the first pair?

(1) (2) (3) (4)

Sol. A quick look at the first pair of objects shows that:


They are of the same shape.
They are of the same size.
They are in the same postion.
The thickness of the lines is the same.
None of these elements can form the link between the objects. So what connection can you see between the two
shapes?

1 line 3 lines
1 line 3 lines
The connection between these two shapes is related to number, each short single line in the first symbol has
become three short lines in the second symbol.
Now you can use this analogy to predict the symbol that will complete the second pair.
1 line 1 line 3 lines 3 lines

1 line 3 lines
This predicted symbol matches option (2).
Hence, the answer is (2).

www.allenoverseas.com OVERSEAS 53
Class-VII

Directions (Q.1 to Q.10) : Each of the following questions consists of two sets of figures. Figures A, B, C and D
constitute the Problem Set while figures 1, 2, 3 and 4 constitute the Answer Set. There is a definite relationship
between figures A and B. Establish a similar relationship between figures C and D by selecting a suitable figure
from the Answer Set that would replace the question mark (?) in fig. (D).
PROBLEM FIGURES ANSWER FIGURES

1.
?
A B C D (1) (2) (3) (4)

2.
?
A B C D (1) (2) (3) (4)

3.
?
A B C D (1) (2) (3) (4)

4.

?
A B C D (1) (2) (3) (4)

5.

?
A B C D (1) (2) (3) (4)

PROBLEM FIGURES ANSWER FIGURES

6.
?
(A) (B) (C) (D) (1) (2) (3) (4)

7.
?
(A) (B) (C) (D) (1) (2) (3) (4)

54 OVERSEAS www.allenoverseas.com
Mental Ability

8.
?
(A) (B) (C) (D) (1) (2) (3) (4)

9.
?
(A) (B) (C) (D) (1) (2) (3) (4)

10.
?
(A) (B) (C) (D) (1) (2) (3) (4)

11. Which shape or pattern completes the second pair in the same way as the first pair?

?
(1) (2) (3) (4)

12. Which shape or pattern completed the second pair in the same way as the first pair?

×
× × ?
×

×
×
×

(1) (2) (3) (4)

13. Which shape or pattern completes the second pair in the same way as the first pair?

?
(1) (2) (3) (4)

www.allenoverseas.com OVERSEAS 55
Class-VII

14. Which shape or pattern completes the second pair in the same way as the first pair?

?
(1) (2) (3) (4)

15. Which shape or pattern completes the second pair in the same way as the first pair?

: :: :?

(1) (2) (3) (4)

Que. 1 2 3 4 5 6 7 8 9 10 11 12 13 14 15
Ans. 2 2 1 2 3 1 1 2 3 1 1 1 1 3 3

56 OVERSEAS www.allenoverseas.com
Mental Ability

CHAPTER 10 NON-VERBAL CLASSIFICATION

In the chapter on Classification, we deal with problems of 'Odd-One-Out' type. In such problems, we are given
a set of figures, such that all, except one have similar characteristics/ features. We are required to select the
figure which differs from all other figures in the given set. Several other types of problems based upon classification
are also discussed in details in this chapter.
Choosing the odd figure
Solved Examples
Direction : In such type of problems, we are given a set of four figures, out of which all except one are alike in
some manner. We have to select the exclusively different figure in the given set. Following examples will make
understanding easier.

Ex.1

(1) (2) (3) (4)


Sol. Clearly, in all other figures. except fig. (4), the two elements on either side of the line are vertically inverted
images of one another.
Hence, the answer is (4).
Ex.2

(1) (2) (3) (4)


Sol. In this case, all the figures, except fig. (4) can be rotated into each other.
Hence, the answer is (4).
Ex.3

(1) (2) (3) (4)


Sol. Only in fig. (3), two of the four elements are oriented in the same direction.
Hence, the answer is (3).
Ex.4

(1) (2) (3) (4)


Sol. In each of the figures, except fig. (3), an odd number of squares are shaded. Hence, the answer is (3).
Ex.5

(1) (2) (3) (4)


Sol The figures form a series. The complete figure rotates 90° CW in each step. Fig (4) does not fit in the series.
Hence, the answer is (4).

www.allenoverseas.com OVERSEAS 57
Class-VII

1. Choose the figure which is different from the rest.

(1) (2) (3) (4)

2. Choose the figure which is different from the rest.

(1) (2) (3) (4)

3. Choose the figure which is different from the rest.

(1) (2) (3) (4)

4. Choose the figure which is different from the rest.

(1) (2) (3) (4)

5. Choose the figure which is different from the rest.

(1) (2) (3) (4)

Directions (Q.6 to Q.10): Out of the given four figures, three are similar in a certain way. One figure is not
like the other three that means three figures to form a group. Which one of the figures does not belong to this
group.

6. (1) (2) (3) (4)

7. (1) (2) (3) (4)

8. (1) (2) (3) (4)

9. Choose the one which is different from the others.

(1) (2) (3) (4)

10. Which is the odd one out?

(1) (2) (3) (4)

58 OVERSEAS www.allenoverseas.com
Mental Ability

Directions (Q.11 to Q.15) : Which figure is the odd one out in the group.

11.

(1) (2) (3) (4)

12.

(1) (2) (3) (4)

13.

(1) (2) (3) (4)

14.

(1) (2) (3) (4)

15.

(1) (2) (3) (4)

Que. 1 2 3 4 5 6 7 8 9 10 11 12 13 14 15
Ans. 1 2 2 3 4 1 3 3 3 1 4 1 4 3 3

www.allenoverseas.com OVERSEAS 59
Class-VII

Important Notes

60 OVERSEAS www.allenoverseas.com
Mental Ability

CHAPTER 11 DIRECTION SENSE TEST


There are four main direction viz. East, West, North and South. East and West as well as North and South are
opposite to each other as shown below.
North (N)

West (W) East (E)

South (S)
The sun always rises in the East and sets in the West.
Four Other Direction: There are four other directions which are also called as sub-main directions, which lie
in between the four main directions.
These are : North-East (N-E); North-West (N-W) ; South-East (S-E); South-West (S-W).
Given below these sub four directions with the main four directions on a plane paper.

Two Cyclic Directions : There are two cyclic directions namely clockwise and anticlockwise. The direction of
moving a clock's hands is called clockwise direction while its opposite direction is called anticlockwise direction as
shown below.

Solved examples
Ex-1. Mohit walks 6 km to the East and turns to the South and walk 5 km. Again he turns to the East and walks 6 km.
Next, he turns northwards and walks 10 km. How far is he from his starting point?
(1) 5 km (2) 12 km (3) 13 km (4) 17 km
Sol. Mohit starts from A walks 6 km East upto B, turns southwards and moves 5 km upto C. At C, he turns to the East
and walks 6 km upto D. He then turns northwards and walks 10 km upto E.
Now, draw BO and AE.
Clearly, BO = CD = 6 km E
AO = (AB + BO) = (6 + 6)km = 12 km
OE = (DE – OD) = (DE–BC) 10 km
= (10–5) km = 5 km B
A O
6 km 5 km
\ Mohit's distance from the starting point A
C D
= AE = AO2 + OE 2 = (12)2 + (5)2 6 km

= 169 =13 km
Hence, the answer is (3).

www.allenoverseas.com OVERSEAS 61
Class-VII

Ex-2. A man is facing North-West. He turns 90° in the clockwise direction and then 135° in the anticlockwise directon.
Which direction is he facing now?
(1) East (2) West (3) North (4) South

Sol.

Turning 90° from NW clockwise means, he is facing NE from NE moving 135° anti-clockwise means he is now
facing West.
So, the man is facing West. Hence, the answer is (2).
Ex-3. A person starts from a point A and travels 3 km eastwards to B and then turns left and travels thrice that
distance to reach C. He again turns left and travels five times the distance he covered between A and B and
reaches his destination D. Find the shortest distance between the starting point and the destination.
(1) 12 km (2) 15 km (3) 16 km (4) 18 km
Sol. The movement of the person is as shown in the figure.
Clearly, AB = 3 km,
West 15km NORTH
BC = 3 AB = (3×3) km = 9 km D E C
CD = 5 AB = (5×3) km = 15 km 12km 3km

Draw AE ^ CD.
Then CE = AB = 3 km and 9km
AE = BC = 9 km
DE = (CD – CE) = (15 – 3) km=12 km.
3km
In DADE, AD2 = AE2 + DE2
A B

Þ AD = ( 2
9 + (12) 2
) km = 225 km = 15 km
East

\ The required distance is AD = 15 km. Hence, the answer is (2).


Ex-4. A watch shows 4:30. If the minute hand points East, in what direction will the hour hand point?
(1) North (2) North-West (3) South-East (4) North-East

W
SW NW

S N
Sol.
NE
SE
E

Hence, the answer is (4).

62 OVERSEAS www.allenoverseas.com
Mental Ability

1. Find the direction which replaces '?' in the following.

(1) N (2) NW (3) SW (4) W


2. Find the direction which replaces '?' in the following.

(1) N (2) NE (3) W (4) SW


3. Find the direction which replaces '?' in the figure below.

(1) NE (2) E (3) SE (4) N


4. Find the direction which replaces '?' in the following figure.

(1) N (2) NE (3) W (4) NW


5. Find the direction which replaces '?' in the figure given below.

(1) S (2) SW (3) SE (4) E


6. Find the direction which replaces '?' in the figure given below.

(1) N (2) NE (3) S (4) SE

www.allenoverseas.com OVERSEAS 63
Class-VII

Directions (Q.7 to Q.14): In the following questions, movements of a rat is given in Column l. Column II shows
the diagrammatic representation of these movements. Match the two columns and write the correct answer in
the given blank grid.
Note:- shows the initial point of the rat in which the arrow shows the face of the rat.
Column I (Movements) Column II (Diagram)

7. right ® right ® left ® left (A)

8. left ® left ® right ® right (B)

9. right ® left ® left ® right (C)

10. left ® right ® right ® left (D)

11. right ® right ® right ® left (E)

12. left ® left ® left ® right (F)

13. left ® right ® right ® right (G)

14. right ® left ® left ® left (H)

Direction (Q.15 to Q.17): In the following diagrams, if CW indicates clockwise and ACW indicates anticlockwise
and arrow indicates the direction of movement. Then fill the appropriate direction in the box.

135°ACW
15.

(1) SW (2) SE (3) S (4) E

16.

(1) N (2) SW (3) E (4) W

64 OVERSEAS www.allenoverseas.com
Mental Ability

17.

(1) SW (2) SE (3) S (4) N


18. 'X' started walking straight towards South. He walked a distance of 5 m and then took a left turn and walked a
distance of 3 m. Then he took a right turn and walked a distance of 5 m again. Which direction 'X' is facing now?
(1) North-East (2) South (3) North (4) South-West

19. A person starts towards South direction. Which of the following orders of direction will lead him to East direction?

(1) Right, Right, Right (2) Left, Left, Left (3) Left, Right, Right (4) Right, Left, Right

20. Darsh started from a point 'A' towards South and travelled 5 km. Then he turned right and travelled 2 km. Then
he turned right and travelled 5 km. Then he turned left and travelled 5 km. How far is he from the point 'A'?

(1) 5 km (2) 7 km (3) 15 km (4) 17 km

Que. 1 2 3 4 5 6 7 8 9 10 11 12 13 14 15 16 17 18 19 20
Ans. 2 1 2 1 2 1 G C E F A B H D 1 2 4 2 1 2

www.allenoverseas.com OVERSEAS 65
Class-VII

1. One morning after sunrise, Rohit was standing facing a pole. The shadow of the pole is exactly to his left. Which
direction was he facing?
(1) East (2) West (3) North (4) South
2. A cat runs 20 m towards East turns right and runs 10 m. She then turns right and runs 9 m. She again turns left
and runs 5 m and then turns left and runs 12 m. Finally she turns left and runs 6 m. Now which direction is the
cat facing?
(1) East (2) North (3) South (4) West
3. You go towards North, turn right, then right again and then go to the left. In which direction are you now?
(1) North (2) South (3) East (4) West
4. From his house, Kabir went 15 km to the North. Then he turned West and covered 10 km. Then, he turned
South and covered 5 km. Finally turning to East, he covered 10 km. In which direction is he from his house?
(1) East (2) West (3) North (4) South
5. Village W is 20 km to the North of village X. Village Y is 18 km to the East of village X. Village Z is 12 km to
the West of W. If Manu starts from village Y and goes to villages Z via village W, in which direction is he from his
starting point?
(1) North-West (2) South (3) North-East (4) East
6. If X is to the South of Y and Z is to the East of Y, in what direction is X with respect to Z?
(1) North-East (2) North-West (3) South-West (4) South-East
7. A watch reads 10:30. If the minute hand points East, in what direction will the hour hand point?
(1) North-East (2) South-West (3) North (4) South
8. If South-East becomes North, North becomes South-West and so on, then what will North-West become?
(1) South-East (2) North-West (3) North (4) South
9. The time on the watch is quarter to three. If the minute hand points to North-East, in which direction does the
hour hand points?
(1) South-East (2) South-West (3) North-East (4) North-West
10. Ankit, Tarun, Rohan and Sohan are friends. They play cards, Ankit and Tarun become partners. Sohan faces
North. If Ankit faces West, then who faces towards South?
(1) Tarun (2) Rohan (3) Sohan (4) Data inadequate
11. Abhishek puts his timepiece on the table in such a way that at 6 pm hour hand points to North. In which direction
the minute hand will point at 9:15 p.m.?
(1) South-East (2) South (3) North (4) West
12. A started from his house towards West. After walking a distance of 45 m, he turned to the right and walked 20
m. He then again turned to the right and walked 45 m. After this he has to turn to his right at 135° and to cover
30 m, In which direction should he go?
(1) West (2) South (3) South-West (4) South-East
13. Four friends A, B, C and D live in a same locality. The house of B is in the East of A's house but in the North of
C's house. The house of C is in the West of D's house. D's house is in which direction of A's house?
(1) South-East (2) North-East (3) East (4) Data inadequate
14. One evening before sunset, Hari was standing facing pole. The shadow of the pole fell exactly to his right. To
which direction was he facing?

(1) East (2) North (3) West (4) Data inadequate

66 OVERSEAS www.allenoverseas.com
Mental Ability

15. A man is facing towards West and turns through 45° clockwise, again 180° clockwise and then turns through
270° anti-clockwise. In which direction is he facing now?

(1) West (2) North-West (3) South (4) South-West

16. l am facing East. l turn 100° in the clockwise direction and then 145° in the anti-clockwise direction. Which
direction am l facing now?

(1) East (2) North-East (3) North (4) South

17. A man is facing North-West. He turns 90° in the clockwise direction, then 180° in anti-clockwise direction and
then another 90° in the anti-clockwise direction. Which direction is he facing now?

(1) South (2) South-West (3) West (4) South-East

18. A girl leaves her house. She first walks 30 metres in North-West direction then 30 metres in South-West
direction. Next she walks 30 metres in South-East direction, Finally, she turns towards her house. In which
direction is she moving now?

(1) North-East (2) North-West (3) South-West (4) South-East

19. A is 40 m South-West of B.C is 40 m South-East of B. then, C is in which direction with respect to A?

(1) East (2) West (3) North-East (4) South

20. One morning after sunrise, Govind's shadow fell exactly to his right. Which direction was he facing?

(1) South (2) East (3) West (4) Date inadequate

Que. 1 2 3 4 5 6 7 8 9 10 11 12 13 14 15 16 17 18 19 20
Ans. 3 2 3 3 1 3 2 4 2 2 4 3 1 2 4 2 4 1 1 1

www.allenoverseas.com OVERSEAS 67
Class-VII

Important Notes

68 OVERSEAS www.allenoverseas.com
Mental Ability

CHAPTER 12 BLOOD RELATION

In this section, problem are based on blood relations. The process of solving these problems (puzzles) depends
upon the deeply knowledge of blood relations. The common blood relations are : Father, Mother Grandparents,
Wife, Husband, Son, Daughter, Grandchild, Sister, Brother etc.
Remarks : 1. Relatives on the mother's side are called 'maternal'. For example, mother's brother is called
maternal uncle.
2. Relatives on the father's side are called 'paternal'. For example, father's brother is called
paternal uncle.
3. Assume a relation as paternal relation, unless stated otherwise.
HOW TO SOLVE PROBLEMS
† To solve problems, find right of required relation and eliminate all wrong choices if any. Also find generation
gaps to solve quickly.
† You can solve by drawing a generation diagram. For this put lower generation below the upper generation and
mention M (or m) and F (or f ) within brackets for male and female respectively.
† Please notice that you can find someone's gender from his/her name.
Solved examples
Ex.1 Pointing to a lady, a man said, "The son of her only brother is the brother of my wife." How is the lady related
to the man ?
(1) Mother's sister (2) Grandmother (3) Mother-in-law (4) Sister of father-in-law
Sol. Brother of my wife - My brother-in-law; Son of lady's brother is the brother-in-law of the man. So lady's brother
is man's father-in-law i.e., the lady is the sister of man's father-in-law.
Hence answer is (4).
Ex.2 If 'X + Y' means X is the father of Y,
'X × Y' means X is the brother of Y,
'X - Y' means X is the mother of Y,
Then which of the following is definitely true about C - A + B?
(1) B is the son of A
(2) A is the son of C
(3) B is the father of A
(4) C is the mother of B
Sol 'C - A' means C is the mother of A,
'A + B' means A is the father of B.
As C is the mother of A and A is the father of B, we can definitely say A is the son of C. Hence answer is (2).
Ex.3 A is father of C and D is son of B. E is brother of A. If C is sister of D, how is B related to E ?
(1) Daughter (2) Husband (3) Brother (4) Sister-in-law
Sol. A is father of C and C is sister of D. So, A is father of D. But D is son of B. So, B is the mother of D and wife of
K Also. E is the brother of A So, B is the sister-in-law of E. Hence answer is (4).

www.allenoverseas.com OVERSEAS 69
Class-VII

1. P is brother of Q, R is mother of P, S is father of R. What is S of Q?


(1) Maternal grandfather (2) Grandson
(3) Uncle (4) Nephew
2. A is brother of B. C is daughter of A.D is son of B. How are C and D related?
(1) Brothers (2) Sisters (3) Cousins (4) Not related
3. If D is the brother of B, how B is related to C?
To answer this question which of the following statement(s) is/are necessary?
1. The son of D is the grandson of C. 2. B is the sister of D.
(1) Only (1) (2) Only (2) (3) Either (1) or (2) (4) Both (1) and (2)
4. P is mother of Q. R is sister of Q. S is sister of P. How is S related to R?
(1) Mother (2) Maternal aunt (3) Nephew (4) Daughter
5. Neeta who is the sister-in-law of Anshu, is the daugher-in-law of Riya. Neeraj is the father of Abhishek who is the
only brother of Anshu. How is Neeta related to Abhishek if Neeraj and Riya are a married couple?
(1) Mother-in-law (2) Aunt (3) Wife (4) None of these
6. Introducing a woman, Sohan said, 'She is the mother of the only daughter of my son". How is that woman related
to Sohan?
(1) Daughter (2) Sister-in-law (3) Wife (4) Daughter-in-law
7. Pointing to a person, Sameer said,"His only brother is the father of my daughter's father." How is the person
related to Sameer?
(1) Father (2) Grandfather (3) Uncle (4) Brother-in-law
8. A is son of B. C is wife of A. D is daughter of C. What is D of B?
(1) Son (2) Daughter (3) Granddaughter (4) Grandson
9. Pointing to Gauri a person said, ' The son of her only brother is the brother of my wife." How is Gauri related to
the person?
(1) Sister-in-law (2) Grandmother (3) Sister of father-in-law (4) None of these
10. Introducing a man, payal said, "He is the only son of the mother of my mother." How is Payal related to the man?
(1) Mother (2) Sister (3) Niece (4) Maternal aunt
Direction (Q.11 to Q.14): Read the information given below and answer the questions that follow.
(i) A, B, C, D, E and F are six members of a family.
(ii) One couple has parents and their children in the familiy.
(iii) A is the son of C and E is the daughter of A.
(iv) D is the daughter of F, who is the mother of E.
11. Who are the male members in the family?
(1) A and C (2) C and F (3) A, B and D (4) Can't be determined
12. Which of the following pairs is the parents of the children?
(1) B and C (2) C and F (3) B and F (4) A and F

70 OVERSEAS www.allenoverseas.com
Mental Ability

13. Which of the following pairs is the parents of the couple?


(1) A and B (2) B and C (3) A and F (4) C and F
14. How many female members are there in the family?
(1) Two (2) Three (3) Four (4) Five
15. A is the mother of B; B is the sister of D; D is the father of M. How is A related to M?
(1) Mother (2) Grandmother (3) Aunt (4) Data inadequate
16. If P is the brother of Q; Q is the sister of R; and R is the father of S, how is S related to P?
(1) Brother (2) Sister (3) Nephew (4) Can't be dermined
17. Pointing to Veer, Veena says, "I am the daughter of the only son of his grandfather." How is Veena related to
Veer?
(1) Niece (2) Daughter (3) Sister (4) Can't be determined
18. Gaurav said to Tarun, "That boy playing with the football is the younger of the two brothers of the daughter of
my father's wife." How is the boy playing football related to Gaurav?
(1) Son (2) Brother (3) Cousin (4) Brother-in-law
19. A and B are children of D. Who is the father of A?
To answer this question which of the following statement(s) (1) & (2) is/are necessary?
1. C is the brother of A and the son of E. 2. F is the mother of B.
(1) Only (1) (2) Only (2) (3) Either (1) or (2) (4) Both (1) and (2)
20. If A $ B' means 'B is the father of A; 'A # B' means 'B is the mother of A'; 'A*B' means 'B is the sister of A' and
'A @ B' means 'B is the husband of A', which of the following indicates that N is the grandmother of P'?

(1) P * Q # M $ N (2) P @ Q $ M # N (3) P # Q $ N * M (4) P * Q $ M # N

21. A's son B is married to C whose sister D is married with E, the brother of B. How is D related to A?
(1) Sister (2) Daughter-in-law (3) Uncle (4) Cousin
22. Pointing to A, B said, " I am the only son of one of the sons of his father". How is A related to B?
(1) Nephew (2) Uncle (3) Father or Uncle (4) Father
23. A photograph shows Mr. P, Mr. Q, Mrs. R and Mrs. S. Mrs. R and Mrs. S are sister. Mr. P is husband of Mrs. R
and Mr. Q is husband of Mrs. S. What is Mrs. R of Mr. Q?
(1) Father-in-law (2) Sister-in-law (3) Son-in-law (4) Daughter-in-law
24. H is husband of N. N is daughter of Mr. W. What is Mr. W of H?
(1) Mother-in-law (2) Sister-in-law (3) Son-in-law (4) Father-in-law

25. Beena, who is Garima's daughter, says to Latika 'your mother Aashka is younger sister of my mother, who is the
third child of Rajeev. How in Rajeev related to Latika?

(1) Maternal Grandfather (2) Uncle

(3) Brother (4) Father

www.allenoverseas.com OVERSEAS 71
Class-VII

Direction (Q.26 to Q.28) : A family consist of six members P ,Q ,R ,S ,T and U. There are two married
couples. Q is a Doctor and the father of T. U is grandfather of R and is a contractor. S is grandmother of T
and is a house wife. There is one Doctor , one Contractor , one Nurse , one Housewife and two students in
the family.
26. Who is the husband of P?
(1) R (2) U (3) Q (4) S
27. Who is the sister of T?
(1) R (2) U (3) Information insufficient (4) None of these
28. What is the profession of P?
(1) Doctor (2) Nurse (3) Housewife (4) None of these

29. If A is the brother of B and K, D is the mother of S and E is the father of A. Which one of the following statements
is definitely not true?

(1) A is the son of D (2) A is the father of K (3) D is the wife of E (4) D is the mother of K

30. A party consist of grandmother , father , mother , four sons and their wives and one son and two daughters
to each of the sons. How many females are there in all?
(1)14 (2)17 (3)16 (4) 24

Que. 1 2 3 4 5 6 7 8 9 10 11 12 13 14 15
Ans . 1 3 4 2 3 4 3 3 3 3 4 4 2 3 2
Que. 16 17 18 19 20 21 22 23 24 25 26 27 28 29 30
Ans . 4 3 2 2 4 2 3 2 4 1 3 3 2 2 1

72 OVERSEAS www.allenoverseas.com
Mental Ability

CHAPTER 13 CUBE & DICE

13.1 Cube
Cuboid : Cuboid is uniform solid which has 6 rectangular faces, 8 corners and 12 edges.

Cuboid

Cube : Cube is a cuboid which has 6 square faces, 8 corners and 12 edges.

Cube
Dice : A dice is a cube which contains 6 different images on its 6 faces.

Dice

Solved example
Ex. Count the number of blocks in the given solid.

(1) 24 (2) 36 (3) 48 (4) 60


Sol. Each layer (say top most) of the solid contains 12 blocks.
There are 3 identical layers in all.
So, the total number of blocks = 3 × 12=36
Hence, the answer is (2).

www.allenoverseas.com OVERSEAS 73
Class-VII

Direction (Q.1 to Q.5) : Count the number of cubes in the given solid in each of the following questions.

1.

(1) 8 (2) 7 (3) 6 (4) 5

2.

(1) 3 (2) 4 (3) 5 (4) 6

3.

(1) 10 (2) 15 (3) 18 (4) 21

4.

(1) 21 (2) 13 (3) 16 (4) 14

5.

(1) 18 (2) 16 (3) 12 (4) 11


6. Which of the following is the net of a cuboid?

(1) (2) (3) (4)

7. How many unit cubes are there in the figure?

(1) 30 (2) 32 (3) 35 (4) 40

74 OVERSEAS www.allenoverseas.com
Mental Ability

8. Number of unit cubes in the given figure is_____.

(1) 25 (2) 26 (3) 27 (4) 28


9. Count the number of cubes in the following figure.

(1) 16 (2) 20 (3) 24 (4) 28


10. Count the number of cubes in the given figure.

(1) 23 (2) 24 (3) 25 (4) None of these

Direction (Q.11 to Q.15) : A solid cube of each side 4 cm, has been painted all faces. If it is then cut
into cubical blocks each of side 2 cm.
11. How many cubes are there in all of a edge 2 cm?
(1) 2 (2) 4 (3) 8 (4) 16
12. How many cubes have no face painted?
(1) 0 (2) 2 (3) 4 (4) 8
13. How many cubes have only one face painted?
(1) 0 (2) 2 (3) 4 (4) 8
14. How many cubes have only two faces painted?
(1) 0 (2) 1 (3) 2 (4) 4
15. How many cubes have only three faces painted?
(1) 2 (2) 4 (3) 6 (4) 8

Que. 1 2 3 4 5 6 7 8 9 10 11 12 13 14 15
Ans. 3 2 3 1 4 4 2 3 3 1 3 1 1 1 4

www.allenoverseas.com OVERSEAS 75
Class-VII

13.2 Dice
Dice is a three dimensional figure with all of its surfaces numbered. We are giving below few properties of Dice
which will help candidates to solve various problems on Dice.
1. A dice has six surfaces and all of them are numbered from 1 to 6.
2. If the surfaces of dice are unfolded and placed on a plane, the figure of dice so obtained will look like one of the
following figures:
Form I In this case

1 1 lies opposite to 5 ;
2 3 4
2 lies opposite to 4 ;
5
6 3 lies opposite to 6.
Form II In this case
1 2 1 lies opposite to 6 ;
3
2 lies opposite to 4 ;
4
5 6 3 lies opposite to 5.
Form III In this case

1 1 lies opposite to 4 ;
2 3 2 lies opposite to 6 ;
4
3 lies opposite to 5.
5 6
Form IV In this case

1 1 lies opposite to 4 ;
2 3
4 5 2 lies opposite to 5:
6
3 lies opposite to 6.

Form V In this case

1 1 lies opposite to 3 ;
2
3 4 2 lies opposite to 5 ;
5
6 4 lies opposite to 6.
Form VI In this case
+ +
– = will be the one of the faces of the cube and it lies opposite to 3;
×
1
– 2 3 4 = 2 lies opposite to 4 ;
5
×
1 lies opposite to 5.

76 OVERSEAS www.allenoverseas.com
Mental Ability

Form VII In this case


+ – +
will be the one of the faces of the cube and it lies opposite to 3;
×=
1

2 3 4 = 2 lies opposite to 4 ;
5
× 1 lies opposite to 5.
Form VIII In this case
1 + =
+ and – × are two faces of the cube that lie opposite to each other..
– 2
1 lies opposite 3 ;
3 =
×
4 2 lies opposite 4 ;
Solved examples
Category–I
Ex.1 A dice has been thrown four times and produces following results. Which number will appear opposite to the number 3?
6 5 3 2
1 4 1 6
3 2 5 3

(i) (ii) (iii) (iv)


(1) 4 (2) 5 (3) 6 (4) 1
Sol. From the figures (i), (ii) and (iv), we find that numbers 6, 1, 5 and 2 appear on the adjacent surfaces to the
number 3. Therefore, number 4 will be opposite to number 3. Hence, the answer is (1)
Category–II
Ex.2 The figures given below show the two different positions of a dice. Which number will appear opposite to
number 2?
4 5
2 6
3 4

(i) (ii)
(1) 3 (2) 4 (3) 5 (4) 6
Sol. The above question, where only two positions of a dice are given, can easily be solved with the following method.
6
IV

5 4 2
V I III

3
II

1
VI
X
Step-I The dice, when unfolded, will appear as shown in Fig.X
Step-II Write the common number of both the dice in the middle block. Since common number is 4, hence
number 4 will appear in the central block.
Step-III Consider the Fig. (i) and write the first number in the anti-clockwise direction of number 4, (common
number) in block I and second number in block II. Therefore, numbers 3 and 2 being the first and
second number to 4 in anti-clockwise directions respectively, will appear in blocks I and II, respectively.
Step-IV Consider Fig. (ii) and write first and second number in the anti-clockwise direction to number 4,
(common number) in block III and IV. Hence, numbers 6 and 5 will appear in the blocks III and IV,
respectively.
Step-V Write the remaining number in the remaining block. Therefore, number 1 will come in the remaining
block.
Now, from the unfolded figures we find that number opposite to 6 is 3, number opposite to 2 is 5 and number
opposite to 4 is 1. Hence, the answer is (3).

www.allenoverseas.com OVERSEAS 77
Class-VII

Category–III
Ex.3 From the following figures of dice, find which number will come in place of '?'.
4 5 1
3 2 ?
2 1 6

(i) (ii) (iii)


(1) 4 (2) 5 (3) 2 (4) 3
Sol. If the above dice is unfolded, it will look like as Fig-I. given below. (Candidate should follow the methods as
explained in the previous example to find the appropriate place of the numbers appearing on the different
surfaces of the dice in the above Fig.)
Now the number in place of '?' can be obtained by making a slight change in the figure as given here.
Now comparing Fig-I. with Fig. (iii) above, we get that number in place of? is 3. Hence, the answer is (4).

4 4
5 2 3 5 2
1 1 3 1 3
6 6
6
(I) (II)
Category–IV
Ex.4 Which of the following dices is identical to the unfolded figure as shown here?
2
5 1
4 3
6

3 1 2 3
4 5 4 4
6 3 6 2

(1) (2) (3) (4)


Sol. From the unfolded figure of dice, we find that number opposite to 2 is 4, for 5 it is 3 and for 1 it is 6. From this
result we can definitely say that figure (2), (3) and (4) can't be the answer figure as numbers lying on the opposite
pair of surfaces are present on the adjacent surfaces. Hence, the answer is (1).

78 OVERSEAS www.allenoverseas.com
Mental Ability

Directions (Q. 1 to Q.5) : In each of the following questions, four positions of the same dice have been shown. You
have to see these figures and select the number opposite to the number as asked in each of the question.
1. Which number is on the opposite surface of number 3?

3 3 2 4
6 1 1 6
5 6 3 1

(i) (ii) (iii) (iv)

(1) 4 (2) 6 (3) 5 (4) 1


2. Which number is opposite to number 5?

3 2 5 6
4 1 4 3
5 6 2 5

(i) (ii) (iii) (iv)


(1) 6 (2) 5 (3) 1 (4) 3
3. Which number is opposite to number 2?

6 4 5 6
4 3 2 5
2 2 3 1

(i) (ii) (iii) (iv)


(1) 4 (2) 6 (3) 1 (4) 3
4. Which letter will be opposite to letter D?

E C D B
C E B A
A D C C

(i) (ii) (iii) (iv)


(1) A (2) B (3) E (4) F
5. Which symbol will come opposite to symbol ¸ ?

–·· –··
= D D =
× = –·· D

(i) (ii) (iii) (iv)


(1) (2) = (3) × (4) D
6. Which number will come opposite to number 2?
5 4
2 2
1 6

(i) (ii)
(1) 5 (2) 1 (3) 6 (4) 3

www.allenoverseas.com OVERSEAS 79
Class-VII

7. Which will be the number at the bottom, if 5 is at the top?

1 6
2 3
4 2

(i) (ii)

(1) 1 (2) 2 (3) 3 (4) 6


8. All the surfaces of the dice contain different numbers in the form of dots. Consider both the figures of dice and
tell how many dots are contained on the face opposite to the face that contains four dots?

(i) (ii)
(1) 2 (2) 3 (3) 5 (4) 6
9. What is the number of dots on the face opposite to the face that contains 2 dots?

(i) (ii)
(1) 1 (2) 3 (3) 4 (4) 6
10. Which alphabet will come on the opposite surface on which alphabet F appears?

C E D
B C A
A D F

(i) (ii) (iii)


(1) B (2) A (3) C (4) E
11. The different positions of a dice has been shown. What digit will be opposite to digit 4?

3 4 2
4 5 3 2 3 1

(1) 6 (2) 1 (3) 5 (4) 2


Directions (Q.12 to Q.14): In each of the following questions, the second die is obtained by rotating the
first die horizontally to right by an angle of 90°. Observe the dice and answer the questions.
12. The faces of this die show numbers 1, 2, 3, 4, 5 and 6. Number on the face opposite to the face showing
4 is

2 2
3 4 5 3

(1) 1 (2) 6 (3) 5 (4) 3

80 OVERSEAS www.allenoverseas.com
Mental Ability

13. The faces of this die show symbols D, #, *, X, p, and O. Symbol on the face opposite to the face showing
o is

D
* # D
o
*

(1) A (2) O or X (3) # (4) X


14. The faces of this die show 1, 2, 3, 4, 5 and 6 dots. Number of dots on the face opposite to the face showing
1 dot is

(1) 4 or 6 (2) 4 (3) 6 (4) 2


Direction (Q.15 to Q.17): In each of the following questions, the second die is obtained by rotating the
first die horizontally to left by an angle of 90°. Observe the dice and answer the questions.
15. The faces of this die show numbers 1, 2, 3, 4, 5 and 6. Number on the face opposite to the face showing
2 is

1 1
2 3 3 4

(1) 3 (2) 6 (3) 4 (4) 5


16. The faces, of this die show symbols D, #, *, X, p, and O. Symbol on the face opposite to the face showing
p is

*
# O *
O o

(1) A (2) # (3) X (4) *


17. The faces of this die show letters A, B, C, D, E and F Letter on the face opposite to the face showing the
letter A is

C
B A C
A E

(1) E (2) D (3) F (4) D or F

www.allenoverseas.com OVERSEAS 81
Class-VII

Direction (Q.18 to Q.20) : In each of the following questions, the second die is obtained by rotating the
first die vertically to right by an angle of 90°. Observe the dice and answer the questions.
18. The faces of this die show numbers 1, 2, 3, 4, 5 and 6. Number on the face opposite to the face showing
1 is

1
2 3 2
5 3

(1) 4 (2) 6 (3) 5 (4) 2


19. The faces of this die show letters A, B, C, D, E and F. Letter on the face opposite to the face showing the
letter B is

A B
B C
D C

(1) E or F (2) E (3) F (4) C


20. The faces of this die show 1 2, 3, 4, 5 and 6 dots. Number of dots on the face opposite to the face showing 6 dots is

(1) 5 (2) 1 or 5 (3) 1 (4) 4


Direction (Q.21 to Q.25) : Choose the box that is similar to the box formed from the given sheet of paper (X).

21.

(X) (1) (2) (3) (4)


(1) 1 and 2 only (2) 2 and 3 only (3) 1 and 4 only (4) 1 and 3 only

22.

(X) (1) (2) (3) (4)


(1) 1 and 2 only (2) 2 and 4 only (3) 2 and 3 only (4) 1 and 4 only

82 OVERSEAS www.allenoverseas.com
Mental Ability

23. =


= =
(X) (1) (2) (3) (4)
(1) 1 only (2) 1 and 3 only (3) 1, 3 and 4 only (4) 1, 2, 3 and 4
F
24. A E
B
C D F F E D
E
B E D BC E
A

(X) (1) (2) (3) (4)


(1) 1 only (2) 2 only (3) 1 and 3 only (4) 1, 2, 3 and 4

25.

(X) (1) (2) (3) (4)


(1) 2 and 3 only (2) 1, 3 and 4 only (3) 2 and 4 only (4) 1 and 4 only

Que. 1 2 3 4 5 6 7 8 9 10 11 12 13 14 15
Ans. 1 3 3 1 1 4 2 4 1 3 2 3 3 1 3
Que. 16 17 18 19 20 21 22 23 24 25
Ans. 2 4 3 1 2 4 3 3 2 2

www.allenoverseas.com OVERSEAS 83
Class-VII

Important Notes

84 OVERSEAS www.allenoverseas.com
Mental Ability

NUMBER, RANKING & TIME


CHAPTER 14 SEQUENCE TEST
Number Test : In problems on Number Test, a sequence of numbers is given and a candidate is asked similar
type of question as asked in Alpha-Numeric Sequence Puzzle.
Solved examples
Ex.1 How many such 5's are there in the following number sequence each of which is immediately preceded by 3 or
4 but not immediately followed by 8 or 9?
35954553584567357554523510
(1) None (2) Three (3) Four (4) Five
Sol. As you know, a number which comes after a given number is said to follow it while the one which comes before
the given number precedes it. Thus, the numbers satisfying the given conditions may be marked as follows:

35954 5 53584 5 673 5 7554 5 23 5 10


Clearly, there are five such 5's. Hence, the answer is (4).
Ranking Test : In Ranking based problems, usually the ranks of one or two persons from the top and from the
bottom are mentioned. A candidate requires either total number of persons or rank of a person.
Solved examples
Ex.2 In a row of girls, Beena is eighth from the left and Priya is seventeenth from the right. If they interchange their
positions, Beena becomes twelfth from the left. How many girls are there in the row?
(1) 25 (2) 24 (3) 29 (4) 28
Sol. Let the point where Priya initially be A. Initially, position of A from the right = 17 th

After interchanging, position of A from the left = 12th


Obviously, the positions of the point A is fixed.
So, number of girls in all = 17+12–1=28
Hence the answer is (4).
Ex.3 In a row of boys facing the North, A is sixteenth from the left end and C is sixteenth from the right end. B, who
is fourth to the right of A, is fifth to the left of C in the row.
How many boys are there in the row?
(1) 39 (2) 40 (3) 41 (4) 42
Sol. Clearly, according to the given conditions, there are 15 boys to the left of A, as well as to the right of C. Also, B
lies between A and C such that there are 3 boys between A and B and 4 boys between B and C.
N

A B C
15 3 4 15
So, number of boys in the row = (15 + 1 + 3 + 1 + 4 + 1 + 15) = 40. Hence, the answer is (2).
Time Sequence Test
To solve the Time based problems the following information are very useful.
— In every 1 hour, the hour hand rotates 30° and the minute hand rotates 360°.
— The month of February has minimum number of days. These are 28 or 29.
— 1 year = 365 or 366 days.
— The year which is divisible by 4 is called a leap year for example year 2012 is a leap year.
— A century which is divisible by 400 is called a leap year for example 2000 is a leap year but 2100 is not.

www.allenoverseas.com OVERSEAS 85
Class-VII

— There are 29 days in February of the leap year.


— There are 366 days in a leap year and 365 days in a non-leap year or ordinary year.
— 1 century = 100 years.
Solved examples
Ex.4 A bus for Delhi leaves every thirty minutes from a bus stand. An enquiry clerk told a passenger that the bus had
already left ten minutes ago and the next bus will leave at 9:35 a.m. At what time did the enquiry clerk give this
information to the passenger?
(1) 9:10 a.m. (2) 8:55 a.m. (3) 9:08 a.m. (4) 9:15 a.m.
Sol. The next bus will leave at 9:35 a.m. This means that the previous bus had left at 9:05 a.m.
But it happened ten minutes before the clerk gave the information to the passenger.
Thus, the enquiry clerk gave the information at 9:15 a.m.
Hence, the answer is (4).
Ex.5 If the seventh day of a month is three days earlier than Friday, what day will it be on the nineteenth day of the
month?
(1) Sunday (2) Monday (3) Wednesday (4) Friday
Sol. As mentioned, the seventh day of the month is three days earlier than Friday, which is Tuesday.
So, the fourteenth day is also Tuesday and thus, the nineteenth day is Sunday.
Hence, the answer is (1).
Ex.6. Satish remembers that his brother's birthday is after fifteenth but before eighteenth of February whereas his
sister Kajal remembers that her brother's birthday is after sixteenth but below nineteenth of February. On which
day in February is Satish's brother's birthday?
(1) 16th (2) 17th (3) 18th (4) 19th
Sol. According to Satish, the brother's birthday is on one of the days among 16th and 17th February.
According to Kajal, the brother's birthday is on one of the days among 17th and 18th February.
Clearly, Satish's brother's birthday is on the day common to both the above groups, i.e. 17th February.
Hence, the answer is (2).

86 OVERSEAS www.allenoverseas.com
Mental Ability

1. How many numbers are there which are divided by 3 and followed by the numbers divided by 4 in this series?
3 4 5 9 6 1 5 3 2 4 5 3 4 3 4 9 2 5
(1) 2 (2) 3 (3) 4 (4) None of these
2. How many 6s in the series are preceded by 5 but not followed by 9?
5 6 8 6 7 6 5 6 5 6 8 5 9 6 5 6 9 6 8 6 5 5 6 8 6 5 9 6 9 5 6 8
(1) 1 (2) 2 (3) 3 (4) 5
3. How many 7's immediately preceded by 6 but not immediately followed by 4 are there in the following series?
74276436753578437672406743
(1) One (2) Two (3) Four (4) Six
4. In the series given below, count the number of 9's, each of which is not immediately preceded by 5 but is
immediately followed by either 2 or 3. How many such 9s are there?
192659383932592934826981926593839
(1) One (2) Four (3) Five (4) Six
5. Bharati is 8 ranks ahead of Divya who ranks twenty-sixth in a class of 42. What is Bharati's rank from the last?
(1) 9th (2) 24th (3) 25th (4) 34th
6. In a row of boys, A is thirteenth from the left and D is seventeenth from the right. If in this row A is eleventh from
the right then what is the position of D from the left?
(1) 6th (2) 7th (3) 10th (4) 12th
7. In a row of forty children, P is thirteenth from the left end and Q is ninth from the right end. How many children
are there between P and R if R is fourth to the left of Q? '
(1) 12 (2) 13 (3) 14 (4) 15
8. Manoj and Sachin are ranked seventh and eleventh respectively from the top in a class of 31 students. What will
be their respective ranks from the bottom in the class?
(1) 20th and 24th (2) 24th and 20th (3) 25th and 21st (4) 26th and 22nd
9. Students line up in a queue in which Ashish stands fifteenth from the left and Sachin is seventh from the right. If
they interchange their places, Sachin would be fifteenth from the right. How many students are there in the
queue?
(1) 21 (2) 22 (3) 29 (4) None of these
10. Nitin was counting down from 32. Sumit was counting upwards, the numbes starting from 1 and he was calling
out only the odd numbers. What common number will they call out at the same time if they were calling out at
the same speed?
(1) 19 (2) 21
(3) 22 (4) They will not call out the same number
11. Priyanka is taller than Shikha and shorter than Sneha. If Sneha is shorter than Yogita, who is the shortest among
them?
(1) Priyanka (2) Shikha (3) Sneha (4) Yogita

www.allenoverseas.com OVERSEAS 87
Class-VII

12. Pratik remembers that his brother's birthday falls after 18th April but before 22nd April while his sister Sakshi
remembers that her brother's birthday falls after 20th April but before 25th April. On what date does her
brother's birthday fall?
(1) 20th April (2) 21st April (3) 19th April (4) 22nd April
13. Mohit went to sleep at 6:45 p.m. If he rose at 12 noon, for how many hours did he sleep?
(1) 17 hours 15 mins (2) 5 hours 15 mins (3) 12 hours (4) 6 hours 45 mins
14. How many 1's are there in the following sequence of numbers which are immediately followed by 2 , if 2 is not
immediately followed by 3?
1 2 1 3 4 5 1 2 3 5 2 1 2 6 1 4 5 1 1 2 4 1 2 3 2 1 7 5 2 1 2 5
(1) 2 (2) 4 (3) 5 (4) 7
Direction (Q.15 & Q.16) : Study the following information and answer the question that follow:
There are 5 boys A, B, C, D and E in a group. A weighs twice as much as D, B weighs four and half times as
C. C weighs half as much as D. D weighs half as much as E. E weighs less than A but more than C.
15. Who is lightest?
(1) A (2) B (3) C (4) D
16. E is heavier than which of the following two boys?
(1) A and C (2) B and C (3) D and A (4) C and D
17. If the second day of a month is a Friday, which of the following would be the last day of the next month which has
31 days?
(1) Sunday (2) Monday (3) Tuesday (4) Data inadequate
18. Saturday was a holiday for Republic Day. 14 th of the next month is again a holiday for Shivratri. what day was
it on the 14th?
(1) Monday (2) Tuesday (3) Thursday (4) Friday
19. X was born on March 6, 1993. The same year Independence Day was celebrate on Friday. On which day was
X born?
(1) Wednesday (2) Thursday (3) Friday (4) Saturday
20. If 30th January 2003 was Thursday, what was the day on 2nd March, 2003?
(1) Tuesday (2) Thursday (3) Saturday (4) Sunday
21. If every second Saturday and all Sunday are holidays in a 30 days month beginning on Saturday, then how many
working days are there in that month?
(1) 20 (2) 21 (3) 22 (4) 23
22. If 25th of August in a year is Thursday, the number of Monday in that month is
(1) 3 (2) 4 (3) 5 (4) 6
23. If 1st october is Sunday, then 1st November will be
(1) Monday (2) Tuesday (3) Wednesday (4) Thursday
24. If 3rd December, 2000 was Sunday, what day was 3rd January, 2001?
(1) Tuesday (2) Wednesday (3) Thursday (4) Friday

88 OVERSEAS www.allenoverseas.com
Mental Ability

25. In the series


641228742153862171413286
How many pairs of alternate numbers have a difference of 2?
(1) 1 (2) 2 (3) 3 (4) 4
26. If it is possible to form a number which is a perfect square of a two digit odd number using the 2 nd, 4th and 7th digit
of the number 739142658 using each only once, Which of the following is at unit place of that two digit odd
number?
(1) 9 (2) 7 (3) 5 (4) 3
27. Among five boys ,Vasant is taller than Manohar, but not as tall as raghu. Jayant is taller than Dutta, but shorter
than Manohar. Who is the tallest in the group?
(1) Vasant (2) Jayant (3) Manohar (4) Raghu
28. If the 7th day of a month is 3 days earlier than Friday, What day will it be on the 19th day of the month?
(1) Monday (2) Sunday (3) Wednesday (4) Friday
29. Muskaan would like to complete all her homework before 10 pm in order to watch an important T.V. programme.
She works 40 mts in each of her five prepared subjects. What is the latest time at which she can start and still
complete her homework in time for the programme?
(1) 7 pm (2) 7:20pm (3) 6:40pm (4) 6:20pm
30. If in the following groups of numbers after adding all the digits of each group, are arranged in decreasing order,
then which will be the middle group?
853, 395, 486, 249, 497, 766, 914
(1) 395 (2) 497 (3) 486 (4) 766

Que. 1 2 3 4 5 6 7 8 9 10 11 12 13 14 15 16 17 18 19 20
Ans. 4 4 2 2 3 2 3 3 3 4 2 2 1 2 3 4 4 3 2 4
Que. 21 22 23 24 25 26 27 28 29 30
Ans. 4 3 3 2 2 1 4 2 3 1

www.allenoverseas.com OVERSEAS 89
Class-VII

Important Notes

90 OVERSEAS www.allenoverseas.com
Mental Ability

FIGURE FORMATION, CONSTRUCTION OF


CHAPTER 15 SQUARES & GROUPING OF IDENTICAL FIGURES

15.1 Figure Formation


This section deals with the following types of problems.
1. Formation of a Figure from its Segments: In such type of problems all the parts to form a figure are
given. A candidate requires to identify the figure so formed out of the four options.
2. Choosing a Pattern which has the same components as a given Pattern: In such type of
problems, a pattern of several components in given. Only one pattern out of four option patterns contains
the same components. A candidate requires to choose such pattern.
Solved examples
Ex.1 Find out which of the figures (1), (2), (3) and (4) can be formed from the pieces given in figure (X).

(X)

(1) (2) (3) (4)

Sol. The parts of figure in option (1) are in figure (X). Hence, the answer is (1).
EX.2 Select that option which has the same components as the given figure (X).

(X)

(1) (2) (3) (4)

Sol. Components of figure (X) and figure (2) are exactly the same. Hence, the answer is (2).

www.allenoverseas.com OVERSEAS 91
Class-VII

15.2 Construction of Squares


Such type of problems are on the basis of the geometrical figures.
A candidate is required to identify the figure which can fit into each other to form a square.
Solved examples
Ex.3 Select a figure from the given four alternatives which fits exactly into figure (X) to form a complete square.

(X)

(1) (2) (3) (4)

Sol. Figures (X) and (3) together form a square.

Hence, the answer is (3).

15.3 Grouping of Identical Figures


In such type of problems a set of some figures is given.
A candidate requires to classify these figures into groups on the basis of some common properties amongst
them.
Solved example
Ex.4 Group the following figures into three classes regarding common properties amongst them.

1 2 3

4 5 6

7 8 9

(1) 4 , 7, 9; 2, 5, 8; 1, 3, 6 (2) 4 , 7, 9; 2, 5, 6; 1, 3, 8
(3) 1 , 2, 3; 4, 5, 6; 7, 8, 9 (4) 1 , 2, 3; 4, 7, 9; 5, 6, 8
Sol. 4, 7, 9 are blank figures.
2, 5, 6 each is divided into 4 parts by two mutually perpendicular lines.
1, 3, 8 each is divided into 6 parts. Hence, the answer is (2).

92 OVERSEAS www.allenoverseas.com
Mental Ability

Directions (Q.1 to Q.3): In each of the following question, find out which of the figures (1), (2), (3) and (4) can
be formed from the pieces given in figure (X).

1. (1) (2) (3) (4)

(X)

2. (1) (2) (3) (4)

(X)

3. (1) (2) (3) (4)

(X)

Direction (Q.4 to Q.7) : In each of the following question, select the option in which all the components of the
figure (X) are available.

4. (1) (2) (3) (4)


(X)

5. (1) (2) (3) (4)

(X)

6. (1) (2) (3) (4)


(X)

7. (1) (2) (3) (4)

(X)

www.allenoverseas.com OVERSEAS 93
Class-VII

Direction (Q.8 to Q.12) Select a figure from the given four alternatives which fits exactly into Figure (X) to
form complete square.

8. (1) (2) (3) (4)


(X)

9. (1) (2) (3) (4)


(X)

10. (1) (2) (3) (4)


(X)

11. (1) (2) (3) (4)


(X)

12. (1) (2) (3) (4)


(X)
Direction (Q.13 to Q.20): In each of the following questions, group the given figures into three classes using
each figure only once. Hence choose the appropriate option.

13.

1 2 3

4 5 6

7 8 9

(1) 7, 8, 9; 2, 3, 4; 1, 5, 6 (2) 1, 2, 3; 4, 5, 7; 6, 8, 9 (3) 1, 6, 8; 3, 4, 7; 2, 5, 9 (4) 1, 6, 9; 3,4,7;2,5,8

14.
1 2 3

4 5 6

7 8 9

(1) 1, 3, 8; 2, 4, 6; 5, 7, 9 (2) 1, 4, 9; 3, 6, 8; 2, 5, 7
(3) 1, 2, 3; 4, 5, 6; 7, 8, 9 (4) 2, 3, 6; 1, 8, 9; 4, 5, 7

94 OVERSEAS www.allenoverseas.com
Mental Ability

15.

(1) 1, 2, 4; 3, 5, 6; 7, 8, 9 (2) 1, 7, 8; 3, 5, 6; 2, 4, 9
(3) 1, 3, 4; 2, 8, 9; 5, 6, 7 (4) 1, 7, 8; 2, 3, 6; 4, 5, 9

16.

(1) 2, 4, 6; 3, 7, 9; 1, 5, 8 (2) 2, 4, 6; 1, 7, 8; 3, 5, 9
(3) 1, 3, 8; 2, 4, 6; 5, 7, 9 (4) 1, 2, 3; 4, 5, 6; 7, 8, 9

17.
1 2 3

4 5 6

7 8 9

(1) 1, 3, 9; 2, 4, 6; 5, 7, 8 (2) 1, 2, 9; 3, 4, 6; 5, 7, 8
(3) 1, 6, 8; 2, 4, 9; 3, 5, 7 (4) 6, 7, 8; 1, 3, 4; 2, 5, 9

18.

1 2 3

4 5 6

7 8 9

(1) 1, 2, 3; 4, 5, 6; 7, 8, 9 (2) 1, 3, 5; 2, 4, 6; 7, 8, 9
(3) 1, 5, 9; 3, 6, 2; 4, 7, 8 (4) 1, 9, 7; 2, 8, 5; 3, 4, 6

www.allenoverseas.com OVERSEAS 95
Class-VII

19.

1 2 3 4

5 6 7

(1) 5, 2, 6; 3, 4, 7; 1 (2) 1, 3; 2, 6 ; 4, 5, 7

(3) 1, 2, 6, 7; 3; 4, 5 (4) 1, 3; 2, 4, 5; 6, 7

20.

1 2 3

4 5 6

7 8 9

(1) 1, 5, 6; 2, 3, 4; 7, 8, 9 (2) 1, 4, 7; 2, 5, 8; 3, 6, 9
(3) 1, 2, 3; 4, 5, 8; 6, 7, 9 (4) 1, 3, 5; 2, 4, 8; 6, 7, 9

Que. 1 2 3 4 5 6 7 8 9 10 11 12 13 14 15 16 17 18 19 20
Ans. 2 4 3 4 3 1 1 4 3 3 4 3 4 2 2 3 1 1 4 1

96 OVERSEAS www.allenoverseas.com
Mental Ability

CHAPTER 16 INSERTING THE MISSING CHARACTER

Chart logic problems have a partially filled in chart or table and expect you to fill it in completely given either the
information in the chart, or some information contained in the question.
Solved examples
Direction : Find the missing character from among the given alternatives.
P G
Ex.1 N
F
22 21

27 15
K E
? J
(1) M (2) P (3) Q (4) S
Sol. Putting A = 1, B = 2, C = 3, D = 4, ............, X = 24, Y = 25, Z = 26, we have:
F + P = 6 + 16 = 22; G + N = 7 + 14 = 21; J + E = 10 + 5 = 15
Since K = 11, so value corresponding to missing letter = (27 – 11) = 16
So, the missing letter is the 16th letter of the English alphabet, which is P. Hence, the answer is (2).

5 12 6 6 21 7 4 8
Ex.2 ?

4 5 10

(1) 22 (2) 34 (3) 32 (4) 36


5´6´4 6´7´5
Sol. Clearly, we have = = 12 ; = 21 ;
10 10
4 ´ 8 ´ 10
So, missing number = = 32
10
Hence, the answer is (3).

13 19 71 9 128 32

Ex.3.
4 10 ?

(1) 10 (2) 15 (3) 20 (4) 25


13 + 19 71 + 9
Sol. Clearly, we have : =4; = 10 ;
8 8
128 + 32
So, missing number = = 20
8
Hence, the answer is (3).

www.allenoverseas.com OVERSEAS 97
Class-VII

Direction (Q.1 to Q.4) : In each of the following questions, a matrix of characters is given. These characters
follows a certain trend, row-wise or column-wise. Find out the trend and choose the missing character from the
given alternatives.

1. (1) 4 6 6 8

5 7 5
(2) 8
4 3 ?
(3) 12
120 126 320
(4) 16

2. (1) 6
3 6 8
(2) 7
5 8 4

(3) 8
4 7 ?

(4) 9

31 17 58 87
3. (1) 5

68 19 61 56
(2) 8
91 22 70 50
(3) 7
10 142 11 ?
(4) 4

4. (1) 88
188 300 263
(2) 96

(3) 238 893 ? 915

(4) 500

5. In the matrix given below, the value of A, B and C are

(1) A = 13, B = 11, C = 9


9 A 12
(2) A = 13, B = 9, C = 11
B 10 7
(3) A = 9, B = 11, C = 13
8 C 11
(4) A = 9, B = 13, C = 11

98 OVERSEAS www.allenoverseas.com
Mental Ability

Direction (Q.6 & Q.7) : In each of these question, which character if placed at the sign of interrogation shall
complete the matrix?
6. (1) N F W O
(2) X
A J K
(3) D
E M ?
(4) P

7. (1) G
A M B N
(2) R
R C S D
(3) T
E U F ?
(4) V

8. The diagram below is a 'magic square' in which all rows and columns and both diagonals add up to 34. Find xy.

(1) 77 1 8 13 12

(2) 60 14

(3) 45 4 x 16 y

(4) 63 15

9. Complete above magic square so that the rows, columns, diagonals-all add to the same number and then find
A+B

3 1
(1) 20 13 4—
4
4
3 A
6 B
(2) 20 4
8 1
5
1 2
(3) 20
4
(4) None of these

Directions (Q.10 to Q.30) : Find the missing character in each of the following questions.

10. (1) 9

(2) 11 1 12 10
15 2 ?
(3) 4
8 5 3
(4) 6

11. (1) 5

(2) 10 6 9 15
8 12 20
(3) 15 4 6 ?
(4) 21

www.allenoverseas.com OVERSEAS 99
Class-VII

12. 93 27 79 38 67 16

63 37 42

3 4 ?

(1) 5 (2) 6 (3) 8 (4) 9


4 8 0
13.
5 0 6 7 1 6 11 ? 2
7 4 2
(1) 0 (2) 2 (3) 11 (4) 12

14. (1) 127

(2) 142 101 43 48 34

38 ?
(3) 158
35 15 56 184
(4) 198

29 27 29 30 59 40

15. 39 72 33 42 73 31 ? 79 10

45 43 43 44 39 20

(1) 49 (2) 50 (3) 60 (4) 69

1 5 4 3 6 3
16. 9 5 8

551 246 ?

3 4 7 9 2 1
6 8 4

(1) 262 (2) 622 (3) 631 (4) 824

17. (1) 10

? 2
(2) 11
9 3
(3) 12 7 4
5 5
(4) 13

18. (1) 33
7
(2) 145 63
2 14 ? 9
(3) 135 30
15
(4) 18

100 OVERSEAS www.allenoverseas.com


Mental Ability

19. (1) 75
2 4 3 9 1 5
(2) 26

(3) 25

(4) 20 20 90 ?

20. (1) 146

(2) 116 5 26 1
9 84 3
(3) 126
11 ? 5
(4) 136
21. (1) 18
7 9 21 27
(2) 24 4 2 36 18
(3) 36 9 4 54 ?
(4) 48
22. (1) 94
4 5 6
(2) 76 2 3 7
(3) 73 1 8 3
21 98 ?
(4) 16
23. (1) 16

(2) 21 84 81 88
(3) 61 14 12 18 9 ? 11
(4) 81
268
24. (1) 72 142 7

(2) 70
16
?
(3) 68 34

(4) 66

25. (1) 4 9
? 3
(2) 5
5 11
(3) 15
7 7
(4) 13 19

26. (1) WJK

(2) KWT Z ? S
R O ?
(3) WKJ ? G C

(4) JKW

www.allenoverseas.com OVERSEAS 101


Class-VII

27. (1) 11

(2) 14 3 6 9
5 8 20
(3) 28 4 7 ?
(4) 12

28. (1) 42

(2) 36 13 54 ?
7 45 32
(3) 6
27 144 68
(4) 4

29. (1) 70
5 9 7
(2) 60 4 5 3
(3) 50 1 6 8
40 100 ?
(4) 80

30. (1) 63
143 3
(2) 56
99 15
(3) 60
? 35
(4) 65

Que. 1 2 3 4 5 6 7 8 9 10 11 12 13 14 15 16 17 18 19 20
Ans. 2 1 3 1 4 3 4 3 4 3 2 4 3 2 4 2 2 3 2 3
Que. 21 22 23 24 25 26 27 28 29 30
Ans. 2 1 1 2 3 3 2 4 3 1

102 OVERSEAS www.allenoverseas.com


Mental Ability

CHAPTER 17 MIRROR & WATER IMAGES

17.1 Mirror Images


The image of an object, as seen in a mirror, is called its mirror reflection or mirror image.
In such an image, the right side of the object appears on the left side and vice - versa. A mirror- image is therefore said
to be laterally inverted and the phenomenon is called Lateral Inversion.
MIRROR-IMAGE OF CAPITAL LETTERS

Letter Mirror Letter Mirror Letter Mirror


Image Image Image

A J S
B K T
C L U
D M V
E N W
F O X
G P Y
H Q Z
I R –

Remark : The letters which have their mirror images identical to the letter itself are:
A, H, I, M, O, T, U, V, W, X, Y
MIRROR-IMAGE OF SMALL LETTERS
Letter Mirror Letter Mirror Letter Mirror
Image Image Image

a j s
b k t
c l u
d m v
e n w
f o x
g p y
h q z
i r –

www.allenoverseas.com OVERSEAS 103


Class-VII

Ex.1 Find the mirror image of the given figure.

(1) (2) (3) (4)

Sol.

Hence, the answer is (3).


Ex.2 Find the mirror image of the given word.
ENGLISH
(1) (2) G (3) H S I L G N E (4)

Sol. ENGLISH

Hence, the answer is (4).


Ex.3 Find the mirror image of the given number. 8193
(1) (2) (3) (4) 3918

Sol. 8193

Hence, the answer is (1).

Ex.4 Find the mirror image of R W

R W R W
(1) (2) (3) (4)

Sol. R W W

Hence, the answer is (3).

104 OVERSEAS www.allenoverseas.com


Mental Ability

Directions (Q.1 to Q.15) : In each of the following questions, you are given a combination of alphabets
followed by four alternatives (1), (2), (3) and (4). Choose the alternative which most closely resembles the mirror
image of the given combination
1. ALLIANCE
(1) (2) (3) (4)
2. CAMPAIGN
(1) (2) (3) (4)
3. commence
(1) (2) (3) (4)
4. doctrine
(1) (2) (3) (4)
5. DoMiNatE
(1) (2) (3) (4)
6. FRANKLIN
(1) (2) (3) (4)
7. IMPERIAL
(1) (2) (3) (4)
8. inherent
(1) (2) (3) (4)
9. IsOLatIOn
(1) (2) (3) (4)
10. MEDIEVAL
(1) (2) (3) (4)
11. OVERHEAD
(1) (2) (3) (4)
12. princess
(1) (2) (3) (4)
13. protocol
(1) (2) (3) (4)
14. RELIANCE
(1) (2) (3) (4)
15. RiGoROUs
(1) (2) (3) (4)
Directions (Q.16 to Q.30) : In each of the following questions, choose the correct mirror-image of the fig. (X)
from amongst the four alternatives (1), (2), (3) and (4) given along with it.
M
16.

(1) (2) (3) (4)

www.allenoverseas.com OVERSEAS 105


Class-VII

17. A

(1) (2) (3) (4)

A
18.

(1) (2) (3) (4)

19.

(1) (2) (3) (4)

A
20.

(1) (2) (3) (4)

106 OVERSEAS www.allenoverseas.com


Mental Ability

M
21.

(1) (2) (3) (4)

M
22.

(1) (2) (3) (4)

23.

(1) (2) (3) (4)

24.

(1) (2) (3) (4)

25.

(X)

(1) (2) (3) (4)

www.allenoverseas.com OVERSEAS 107


Class-VII

26.

(X)

(1) (2) (3) (4)

27.

(X)

(1) (2) (3) (4)

28.

(X)

(1) (2) (3) (4)

29.

(X)

(1) (2) (3) (4)

30.

(X)

(1) (2) (3) (4)

Que. 1 2 3 4 5 6 7 8 9 10 11 12 13 14 15 16 17 18 19 20
Ans. 3 1 4 1 4 1 2 4 2 4 1 3 2 1 4 2 4 1 3 2
Que. 21 22 23 24 25 26 27 28 29 30
Ans. 4 2 3 2 3 2 2 1 3 3

108 OVERSEAS www.allenoverseas.com


Mental Ability

17.2 Water Images


The reflection of an object, as seen in water, is called its water image. It is the inverted image obtained by turning
the object upside down.
WATER-IMAGE OF CAPITAL LETTERS

Letter

Water
Image

Letter

Water
Image
Letter

Water
Image
Remark 1 : The letters whose water-images are identical to the letter itself are:
C, D, E, H, I, K, O, X.
Remark 2 : Certain words which have water-images identical to the word itself are:
KICK, KID, CHIDE, HIKE, CODE, CHICK
WATER-IMAGE OF SMALL LETTERS

Letter
Water
Image
Letter
Water
Image
Letter
Water
Image
WATER-IMAGE OF NUMBERS

Number 9
Water
Image

Solved example
Ex. Choose the Water image of the given word.
G7R18E5E5N14
(1) (2) (3) (4)

Sol.

Hence, the answer is (4).

www.allenoverseas.com OVERSEAS 109


Class-VII

Directions (Q.1 to Q.15) : In each of the following questions, you are given a combination of alphabets
followed by four alternatives (1), (2), (3) and (4). Choose the alternative which most closely resembles the water
image of the given combination

1. STERLING

(1) (2) (3) (4)

2. SUBURBAN

(1) (2) (3) (4)

3. tactical

(1) (2) (3) (4)

4. treasury

(1) (2) (3) (4)

5. UnLaWfUl

(1) (2) (3) (4)

6. ViOlEnCe

(1) (2) (3) (4)

7. WeGhiteD

(1) (2) (3) (4)


8. relegate

(1) (2) (3) (4)

9. DILEMMA

(1) (2) (3) (4)

10. DOCILE

(1) (2) (3) (4)

11. exhaustive

(1) (2) (3) (4)

12. ChAoTiC

(1) (2) (3) (4)

13. oPPreSS

(1) (2) (3) (4)

110 OVERSEAS www.allenoverseas.com


Mental Ability

14. LUNACY

(1) (2) (3) (4)

15. infamous

(1) (2) (3) (4)


Directions (Q.16 to Q.30) : In each of the following questions, choose the correct water-image of the fig. (X)
from amongst the four alternatives (1), (2), (3) and (4) given along with it.

16.

(X)

(1) (2) (3) (4)

17.

(X)

(1) (2) (3) (4)

18.

(X)

(1) (2) (3) (4)

19.

(X)

(1) (2) (3) (4)

www.allenoverseas.com OVERSEAS 111


Class-VII

20.

(X)

(1) (2) (3) (4)

21.

(X)

(1) (2) (3) (4)

22.

(X)

(1) (2) (3) (4)

23.

(X)

(1) (2) (3) (4)

112 OVERSEAS www.allenoverseas.com


Mental Ability

24.

(X)

(1) (2) (3) (4)

25.

(X)

(1) (2) (3) (4)

26.

(X)

(1) (2) (3) (4)

27.

(X)

(1) (2) (3) (4)

www.allenoverseas.com OVERSEAS 113


Class-VII

28.

(X)

(1) (2) (3) (4)

29.

(X)

(1) (2) (3) (4)

30.

(X)

(1) (2) (3) (4)

Que. 1 2 3 4 5 6 7 8 9 10 11 12 13 14 15 16 17 18 19 20
Ans. 2 4 1 3 1 4 1 2 3 4 1 2 4 1 3 3 1 2 4 2
Que. 21 22 23 24 25 26 27 28 29 30
Ans. 4 1 3 2 1 3 1 2 1 3

114 OVERSEAS www.allenoverseas.com


Mental Ability

ALPHABET TEST
CHAPTER 18
18.1 Alphabetical order of words
Alphabetical order: Arranging words in alphabetical order implies 'to arrange them in the order as they
appear in a dictionary', i.e., as per the order in which the beginning letters of these words appear in the English
alphabet.
How to arrange the words in alphabetical order?
First consider the first letter of each word. Arrange the words in the order in which these letters appear in the
English alphabet.
Example : Consider the words - Abstract, Principle, Marry, Spring, Frequent.
These words begin with letters A, P, M, S, F respectively.
Their order in English alphabet is A, F, M, P, S.
Thus, the correct alphabetical order of these words is :
Abstract, Frequent, Marry, Principle, Spring
In some cases, two or more words begin with the same letter. Such words should be arranged in the order of
second letters in the alphabet.
Example : Consider the words - Client, Castle, Face, Viper, Dazzle.
Here, as in the above example, the words can be arranged as :

Cli ent ü
ý , Dazzle, Face, Viper
Castle þ

What remains now is how to arrange 'Client' and 'Castle'. Consider their second letters, i,e., l and a. Arranging
these words accordingly, 'Castle' comes before 'Client'.
Thus, the correct alphabetical order is :
Castle, Client, Dazzle, Face, Viper
If both the first and second letters of two or more words are the same, arrange these words, considering their
third letters and so on.
Solved examples
Ex.1 If the following four words are arranged in alphabetical order, which word will comes at third place?
(1) Draw (2) Read (3) Play (4) Back
Sol. Arranging the words in alphabetical order, we have: Back, Draw, Play, Read.
Clearly, the word 'Play' comes at third place. Hence, the answer is (3).
Ex.2 Which of the following words will come fourth in the English dictionary?
(1) False (2) Follow (3) Faithfully (4) Fallible
Sol. The given words can be arranged in the alphabetical order as :
Faithfully, Fallible, False, Follow.
Clearly, 'Follow' comes fourth. Hence, the answer is (2).

www.allenoverseas.com OVERSEAS 115


Class-VII

Ex.3 Which of the following words will come second in the English dictionary?
(1) Magical (2) Magnify (3) Maternal (4) Magnetic
Sol. The given words can be arranged in the alphabetical order as :
Magical, Magnetic, Magnify, Maternal
Clearly, 'Magnetic' comes second. Hence, the answer is (4).
Ex.4 If the letters in each of the following four words are first rearranged in the alphabetical order and then the
groups of letters so formed are rearranged as in a dictionary, which word would have its group of letters at third
place?
(1) Road (2) Deaf (3) Code (4) Lack
Sol. The letter-groups obtained on rearranging the letters of the given words in alphabetical order are: ador, adef,
cdeo, ackl.
These letter-groups may be arranged in alphabetical order as : ackl, adef, ador, cdeo.
The letter-group 'ador' comes at third place and the word corresponding to this letter-group is 'road'.
Hence, the answer is (1).
Ex.5. Arrange the following words according to dictionary arrangement :
1. Epitaxy 2. Episode 3. Epigene 4. Epitome 5. Epilogue
(1) 1, 2, 3, 4, 5 (2) 3, 2, 5, 4, 1 (3) 3, 5, 2, 1, 4 (4) 5, 4, 2, 1, 3
Sol. Clearly, the correct alphabetical order of the given words is :
Epigene, Epilogue, Episode, Epitaxy, Epitome
Thus, the correct sequence is 3, 5, 2, 1, 4. Hence, the answer is (3).

Directions (Q.1 to Q.5) : Arrange the given words in the alphabetical order and tick the one that comes last.
1. (1) Heredity (2) Hesitate (3) Heavy (4) Hedge
2. (1) Warring (2) Waving (3) Watching (4) Waiting
3. (1) Illiterate (2) Illusion (3) Illuminate (4) Illustrate
4. (1) External (2) Extraordinary (3) Extraction (4) Extremist
5. (1) Transmit (2) Transplant (3) Transport (4) Translate
Directions (Q.6 & Q.7) : In each of the following questions, arrange the given words in the sequence in which
they occur in the dictionary and then choose the correct sequence.

6. 1. Pestle 2. Pestilence 3. Pester 4. Pest


(1) 4, 3, 2, 1 (2) 4, 3, 1, 2 (3) 3, 4, 2, 1 (4) 4, 1, 2, 3
7. 1. Intrinsic 2. Intrude 3. Intricate 4. Introvert 5. Intrigue 6. Introduce
(1) 3, 5, 1, 4, 6, 2 (2) 3, 5, 1, 6, 4, 2 (3) 3, 1, 5, 4, 6, 2 (4) 5, 1, 3, 2, 4, 6

116 OVERSEAS www.allenoverseas.com


Mental Ability

8. If the first five words in the sentence, "Meeta's mother meets me many times" are rearranged in the alphabetical
order, which will be the third word?
(1) Meeta (2) mother (3) meets (4) me
Directions (Q. 9 & Q.10) : In each of the following questions, arrange the given names in the order in which
they would occur in a telephone directory and choose the one which appears third.
9. (1) Avdesh (2) Avadhesh (3) Awadesh (4) Awdhesh
10. (1) Randhir (2) Randesh (3) Rama (4) Raamesh
11. The position of how many letters in the word BRAKES remains unchanged when they are arranged in alphabetical
order?
(1) One (2) Two (3) Three (4) More than three
12. If the letters of the word TRANSFORM are rearranged as they appear in the English alphabet, then the position
of how many letters will remain unchanged after such rearrangement?
(1) One (2) Two (3) Three (4) Four
13. The positions of how many letters in the word WONDERFUL will remain unchanged when the letters within the
word are arranged alphabetically?
(1) None (2) One (3) Two (4) Three
14. Which letters in the word CYBERMETICS occupies the same position as it does in the English alphabet?
(1) B (2) E (3) C (4) T
15. If the first and second letters in the word DEPRESSION were interchanged, also the third and the fourth letters,
the fifth and the sixth letters and so on, which of the following would be the eighth letter from the left?
(1) R (2) O (3) S (4) I
16. Arrange the given words in Alphabetical order and choose the one that comes last.
(1) Wasp (2) Waste (3) War (4) Wrinkle
17. Some letters are given which are numbered 1, 2, 3, 4, 5 and 6. Select that combination of numbers for which
arrangement of letters form a meaningful word,
R F WE L O
1 2 3 4 5 6
(1) 2, 5, 6, 3, 1, 4 (2) 2, 5, 6, 3, 4, 1 (3) 2, 5, 6, 4, 3, 1 (4) 2, 5, 6, 1, 3, 4
18. Arrange the given words in the alphabetical order and tick the one that comes last.
(1) Nozzle (2) Nausea (3) Nostril (4) Nomenclature
19. Arrange the given words in the sequence in which they occur in the dictionary.
1. Shroud 2. Shudder 3. Shuffle 4. Shuttle
5. Shrub
(1) 1,2,3,4,5 (2) 3,2,5,4,1 (3) 1,5,2,3,4 (4) 1,5,3,2,4
20. How many such pairs of letters are there in the word PREDICTABLE which have as many letters between them
in the word as in the English alphabet?
(1) 2 (2) 4 (3) 6 (4) 8

Que. 1 2 3 4 5 6 7 8 9 10
Ans . 2 2 4 4 3 1 2 1 3 2
Que. 11 12 13 14 15 16 17 18 19 20
Ans . 2 1 2 2 3 4 2 1 3 2

www.allenoverseas.com OVERSEAS 117


Class-VII

18.2 Alphabetical quibble


In this type of questions, generally a letter-series is given, be it the English alphabets from A to Z or a randomised
sequence of letters. The candidate is then required to trace the letters satisfying certain given conditions as
regards their position in the given sequence or the sequence obtained by performing certain given operations on
the given sequence.

Solved Examples

Ex.1 Answer the given questions based on the following English alphabet:

ABCDEFGHIJKLMNOPQRSTUVWXYZ

1. Which letter is fifth to the right of the eighteenth letter from your right?

(1) C (2) D (3) E (4) N

2. If all the vowels are removed from the alphabet, which letter will be seventh to the right of the fifth letter from
the left?

(1) L (2) M (3) N (4) P

3. If every alternate letter, starting with A, is removed from the alphabet, which letter among the remaining letters
would be the third to the right of the fifth letter from the right?

(1) X (2) V (3) L (4) J

4. If A and B are written as B and A, C and D as D and C, E and F as F and E and so on till the end of the alphabet,
what will be the position of M from your right?

(1) 11th (2) 12th (3) 13th (4) 14th

5. If the above alphabet is arranged in reverse order, which letter will be twelfth to the left of the sixteenth letter
from your left?

(1) D (2) V (3) W (4) X

Sol.

1. Counting from the right end of the given alphabet-series, i.e. from Z, the eighteenth letter is I. Counting from I
towards the right, the fifth letter is N. .

Hence, the answer is (4).

2. Removing all the vowels, i.e., A, E, I, O, U, we obtain the following letter-series:

BCDFGHJKLMNPQRSTVWXYZ

Counting from the left in the above series, i.e. from B, the fifth letter is G. Counting from G towards right, the
seventh letter is P.

Hence, the answer is (4).

118 OVERSEAS www.allenoverseas.com


Mental Ability

3. Leaving out A, C, E, G, ..... , W, Y, we obtain the following letter-series:

BDFHJLNPRTVXZ

Counting from the right in the above letter-series, the fifth letter is R. Counting from R, the third letter to the
right is X. Hence, the answer is (1).

4. The new letter-series obtained is :

BADCFEHGJILKNMPORQTSVUXWZY

Counting from Y in the above series, M is the 13 th letter. Hence, the answer is (3).

5. We obtain the following letter-series on reversing the order of the alphabets:

ZYXWVUTSRQPONMLKJIHGFEDCBA

Counting from left, i.e. from Z in the above series, the sixteenth letter is K. Counting from K towards left, the
twelfth letter is W. Hence, the answer is (3).

Ex.2. How many D's are there in the following series which are immediately followed by W but not immediately
preceded by K?

KDCWKDWNKGDWWDHKVDWZDW

(1) Nil (2) One (3) Two (4) Three

Sol. Clearly, D's satisfying the given conditions can be marked as under:

KDCWKDWNKG D WWDHKV D WZ D W

We observe that such D's are three in number. Hence, the answer is (4).

Ex.3. If each letter in the English alphabet is attached a value equal to its serial number in the alphabet, which among
the following will have the highest sum of the values of all its letters?

(1) WIND (2) TONE (3) BUDY (4) MORE

Sol. Putting A = 1, B = 2, C = 3, D = 4, ..... , Z = 26, we may calculate the sum of the values of the letters of each
of the given words as follows:

(1) WIND ® W + I + N + D = 23 + 9 + 14 + 4 = 50

(2) TONE ® T + O + N + E = 20 + 15 + 14 + 5 = 54

(3) BUDY ® B + U + D + Y = 2 + 21 + 4 + 25 = 52

(4) MORE ® M + O + R + E = 13 + 15 + 18 + 5 = 51

The value is highest in the case of TONE. Hence, the answer is (2)

www.allenoverseas.com OVERSEAS 119


Class-VII

Directions (Q.1 to Q.5) : Each of the following questions is based on the following alphabet-series:

ABCDEFGHIJKLMNOPQRSTUVWXYZ

1. Which letter is exactly midway between G and Q in the given alphabet?

(1) K (2) L (3) M (4) N

2. Which letter is midway between the eighteenth letter from the left end and tenth letter from the right end of the
given alphabet?

(1) No letter (2) K (3) Q (4) R

3. Which letter in the alphabet is as far from G as T is from M?

(1) M (2) N (3) O (4) P

4. Which letter is sixteenth to the right of the letter which is fourth to the left of I?

(1) S (2) T (3) U (4) V

5. Which letter will be eighth to the right of the third letter of the second half of the English alphabet?

(1) V (2) W (3) X (4) Y

6. If in the English alphabet every fourth letter is replaced by the symbol (t), which of the following would be
seventh to the left of the fourteenth element from the left?

(1) G (2) H (3) T (4) t

7. If 1st and 26th, 2nd and 25th, 3rd and 24th, and so on, letters of the English alphabet are paired, then which of the
following pairs is correct?

(1) GR (2) CW (3) IP (4) EV

8. If every alternative letter of English alphabet from B onwards (including B) is written in lower case (small letters)
and the remaining letters are capitalized, then how will the first month of the second half of the year be written?

(1) JuLy (2) AuGuSt (3) jUlY (4) AugUSt

9. If every alternate letter starting from B of the given alphabet is written in small letters, rest all are written in
capital letters, how will the month of 'September' be written?

(1) SEpteMbeR (2) SEptembER (3) sePTemBeR (4) SEptEMbEr

10. If the alternate letters in the given alphabet starting from A are written in small and rest all in capital letters,
which of the following will represent the third month after July?

(1) OCTOBER (2) OCtObEr (3) oCtObEr (4) ocToBeR

11. How many A's are there in the following series which are immediately followed by B as well as immediately
preceded by Z?

AMBZANAABZABAZBAPZABAZAB

(1) Nil (2) One (3) Two (4) Three

120 OVERSEAS www.allenoverseas.com


Mental Ability

12. In the given series of letters, how many t's are preceded and followed by t?

PtPttPPtPtPPPqqPtPttPPPt

(1) 0 (2) 2 (3) 3 (4) 4

13. In the following list of letters, how many O's are followed by Q's but not preceded by D's?

DOQODQODODQDOQDSDQPOQDSSSDOQOQDOQDDDOQ

(1) 0 (2) 1 (3) 2 (4) 3

14. How many T's are there in the following sequence which are immediately preceded by P but not immediately
followed by S?

STPQTSPTRPTSRPSTQPTRPTMPTS

(1) None (2) One (3) Two (4) Three

15. How many L's are there which do not have R preceding them and also do not have T following them?

ZQSTLRMNQNRTUVXRLTASLTQRSLT

(1) 1 (2) 2 (3) 3 (4) 5

16. If the alphabet series is arranged in reverse order, which letter will be twelfth to the left of the fourteenth letter
from your left?

A B C D E F G H I J K L M N O P Q R S T U V W X Y Z

(1) Y (2) W (3) M (4) D

17. How many a's are there in the odd places in the sequence?

a a a a b a b a b a b b a a a b b a a a

(1) 5 (2) 6 (3) 7 (4) 8

18. How many b's are there in the even places in the sequence?

a a c d j d j d b b bb d s b b s b b s b s

(1) 7 (2) 6 (3) 8 (4) 4

19. What is the letter which is two places to the right of the letter which is third letter to the Left of the letter M in
English alphabet?

(1) J (2) L (3) H (4) N

20. Which is the twelfth letter to the left of the seventeenth letter from the left end in English alphabet?

(1) E (2) F (3) G (4) T

21. Find the letter which is 2nd to the right of the letter which is 5th to the right of M in English alphabet.

(1) T (2) O (3) S (4) R

www.allenoverseas.com OVERSEAS 121


Class-VII

22. Which letter should be ninth letter to the left of ninth letter from the right, if the first half of the given alphabet
is reversed?

(1) D (2) E (3) F (4) I

23. What should come next in the letter series given below:

A A B A B C A B C D A B C D E A B C D E F A B C D E F G A B C D E F G __________.

(1) A (2) I (3) H (4) B

24. Which letter is tenth to the right of the letter which is exactly the middle letter between F and D?
FJMPOWRNBEYCKAVLDGXUHQISZT
(1) D (2) G (3) H (4) None of these
25. Which of the following letters in the word 'SELFRIGHTEOUSNESS' does not change its position when the letters
are reversed?

(1) E (2) G (3) H (4) T

Que. 1 2 3 4 5 6 7 8 9 10 11 12 13 14 15 16 17 18 19 20
Ans . 2 1 2 3 3 1 4 3 4 4 4 1 3 4 1 1 1 4 2 1
Que. 21 22 23 24 25
Ans . 1 2 3 4 4

122 OVERSEAS www.allenoverseas.com


Mental Ability

PAPER FOLDING
CHAPTER 19 & PAPER CUTTING
19.1 Folding a Transparent Sheet
Spotting
In such type of problemsOut the sheet
a figure of transparent Embedded
carrying a design on it is given.Figures
There is a dotted line on
this sheet. This sheet has to be folded along the dotted line.
A candidate requires to identify a figure from given options, that looks similar to the folded sheet.
Solved example
Ex.1 A square transparent sheet X, with a design and a dotted line on it is given. Choose the correct figure from the
options which represents the sheet X after folding sheet along the dotted line.

(1) (2) (3) (4)

Sol. Clearly, the right half of the sheet X is put on the left half. The combination of the design in left half and mirror
image of the design in the right half will appear on the folded sheet. So the sheet will then appear as shown in
figure (3). Hence, the answer is (3).

19.2 Cutting/Punching a Folded Paper


In such type of problems a paper is folded twice or more than twice. Then one or more pieces of it are cut. After
this the paper is unfolded. In this sitution the paper has as many cuts or holes on it as folded. So it contains a
pattern.
A candidate requires to identify a figure from given four options, that shows the similar paper sheet as the
pattern made.
Usually, the paper sheet is folded along the dotted lines marked on it. And arrows show the directions of the folds.
Solved example
Ex.2 Figures X and Y respectively shows the two consecutive folds of the paper. Figure Z shows the cut on the folded
paper. Choose one figure from the four options that is similar to the unfolded form of the sheet Z.

X Y Z

(1) (2) (3) (4)

Sol. In figure (X), the square sheet of paper has been folded along the vertical line of symmetry so that the right half
of the sheet overlaps the left half.
In figure (Y), the sheet is folded further to a quarter.
In figure (Z), a square has been punched in the folded sheet Y.
Clearly, the punched square will be created in each quarter of the paper.
Thus, when the paper in unfolded, four square punches will appear symmetrically over it and the paper will
then appear as shown in figure (4).
Hence the answer is (4).

www.allenoverseas.com OVERSEAS 123


Class-VII

Directions (Q.1 to Q.17): In each of the following questions a transparent sheet X with a design and a dotted
line on it is given. This sheet X is to be folded along dotted line. Choose the correct figure from the four
alternatives, that is similar to the folded form of the sheet X.

1. (1) (2) (3) (4)

(X)

2. (1) (2) (3) (4)


(X)

3. (1) (2) (3) (4)


(X)

4. (1) (2) (3) (4)


(X)

5. (1) (2) (3) (4)


(X)

6. (1) (2) (3) (4)


(X)

7. (1) (2) (3) (4)


(X)

8. (1) (2) (3) (4)


(X)

9. (1) (2) (3) (4)


(X)

10. (1) (2) (3) (4)

124 OVERSEAS www.allenoverseas.com


Mental Ability

11. (1) (2) (3) (4)


(X)

12. (1) (2) (3) (4)


(X)

13. (1) (2) (3) (4)


(X)

14. (1) (2) (3) (4)


(X)

15. (1) (2) (3) (4)


(X)

16. (1) (2) (3) (4)


(X)

17. (1) (2) (3) (4)


(X)
Direction (Q.18 to Q.30) : There are three forms X, Y and Z of a sheet. Figures X and Y respectively show
the two consecutive folds of the sheet. And the figure Z shows punch on the folded sheet. Choose one figure
from the four options (1), (2), (3) and (4), that is similar to the unfolded form of the sheet?

18. (1) (2) (3) (4)


X Y Z

19. (1) (2) (3) (4)


X Y Z

20. (1) (2) (3) (4)


X Y Z

21. (1) (2) (3) (4)

www.allenoverseas.com OVERSEAS 125


Class-VII

22. (1) (2) (3) (4)


X Y Z

23. (1) (2) (3) (4)


X Y Z

24. (1) (2) (3) (4)

25. (1) (2) (3) (4)


X Y Z

26. (1) (2) (3) (4)

27. (1) (2) (3) (4)


X Y Z

28. (1) (2) (3) (4)


X Y Z

29. (1) (2) (3) (4)


X Y Z

30. (1) (2) (3) (4)


X Y Z

Que. 1 2 3 4 5 6 7 8 9 10 11 12 13 14 15 16 17 18 19 20
Ans. 1 3 3 4 2 4 1 3 2 4 2 3 2 4 1 4 1 3 3 4
Que. 21 22 23 24 25 26 27 28 29 30
Ans. 1 3 3 4 2 4 2 2 4 2

126 OVERSEAS www.allenoverseas.com


Mental Ability

ALPHA-NUMERIC SEQUENCE
CHAPTER 20 PUZZLE & LOGICAL SEQUENCE OF WORDS

20.1 Alpha-Numeric Sequence Puzzle


In this type of questions, a jumbled sequence of some letters, numbers and symbols is given, followed by certain
questions based on it.

The questions asked may be any of the following types :

1. To find elements (letters, numbers or symbols) satisfying certain conditions in the given sequence.

2. To determine the element at a certain given relative position in the sequence.

3. To complete a series consisting of terms formed by a combination of the elements of the sequence, by
deciphering the hidden pattern in relation to the position of the elements in the sequence.

4. To find the odd combination from among a given set of combinations of elements, by finding the common
property which other combinations share as per their positions in the given sequence.

5. To find the analogical relationship between a given pair of combinations of elements, and then find a
combination which holds same relationship with another given combination.

Solved examples

Study the following arrangement of symbols, letters and numbers to answer the questions given below it :

d= b F 2 K S 7 5 # $ P L V 8 @ M U E 6 ­ Q G © 9 3 & T Y £

Ex.1 How many such consonants are there in the above arrangement each of which is immediately followed by
a consonant but not immediately preceded by a symbol?

(1) Nil (2) One (3) Two (4) Three

Sol. We know that of the 26 letters of English alphabet, five letters namely A, E, I, O, U are vowels, while remaining
are consonants.

The consonants satisfying the given conditions may be shown as under :

d= b F 2 K S 7 5 # $ P L V 8 @ M U E 6 ­ Q G © 9 3 & T Y £

Clearly, there is only one such consonant.

Hence, the answer is (2).

Ex.2 Which of the following is exactly in the middle of the ninth from the left end and fifteenth from the right
end in the above arrangement?

(1) P (2) U (3) E (4) L

Sol. Clearly, counting from the left, the ninth element is 7. Counting from the right, the fifteenth element is @.
The character midway between 7 and @ is P.

Hence, the answer is (1).

www.allenoverseas.com OVERSEAS 127


Class-VII

Ex.3 Which of the following groups of elements will come in place of the question mark in the series of elements
given below?
K 2 d 5 S F P # K ? U @ P
(1) 8 L 5 (2) @ V # (3) V P 7 (4) 8 $ 5
Sol. Clearly, first and second elements of each term move two steps and four steps backward respectively to give
the second and third elements of the same term. The third element of each term moves nine steps forward
to give the first element of the next term. The pattern is repeated in the subsequent terms.
Now, the letter 9 steps ahead of K is 8; the letter 2 steps behind 8 is L and the letter 4 steps behind L
is 5. Thus, the missing term is 8 L 5. Hence, the answer is (1).
Ex.4 Based on the positions in the above arrangement, if b F d : T & £, then K 7 :?
(1) © 9 Q (2) © G Q (3) 9 © Q (4) © 9 ­
Sol. Observing the given analogous pair, we find that the corresponding elements of both the terms occupy the
same position from the beginning and end of the given arrangement.
Now, K, and and 7 are respectively 7th, 6th and 9th elements from the left end in the given arrangement.
The 7th, 6th and 9th elements from the right end are ©, 9 and Q respectively. Thus, the missing term is
© 9 Q. Hence, the answer is (1).

Direction (Q.1 to Q.5) : Study the following arrangement carefully and answer the questions given below :
8 C M @ N £ T 2 Y 6 S a Q $ 7é W # Z 3 U E % A 4
1. How many symbols are there in the above series each of which is immediately preceded and also immediately
followed by a vowel?
(1) Nil (2) One (3) Two (4) Three
2. If all the vowels are dropped from the above series, which of the following would be the eighth element to the
right of the thirteenth element from the left end?
(1) 4 (2) 8 (3) % (4) C
3. If each symbol is first converted into a numeral and then all the numerals are converted into English letters, how
many converted English letters will be there in the above arrangement of elements?
(1) 7 (2) 12 (3) 13 (4) 25
4. What should come in place of the question mark in the following series based on the above arrangement?
CMA N£E 2Y3 ? $7é
(1) £ Q # (2) S a # (3) S 6 a (4) S a Z
5. Three of the following four are alike in a certain way with respect to their position in the above arrangement.
Which is the one, that is different from the other three?
(1) £ 2 6 (2) Q 7 W (3) E 3 # (4) T M N
Directions (Q.6 to Q.9): Study the following arrangement of letters, digits and symbols to answer the ques-
tions given below :
A B 7 C D 9 Z Yé P 2 M © K S 3 ­ 5 N T @
6. Which of the following letters is exactly midway between only letters falling between C and 5?
(1) K (2) M (3) P (4) Y

128 OVERSEAS www.allenoverseas.com


Mental Ability

7. If each symbol of the above sequence is replaced with a letter and each digit is replaced with new symbol, then
how many letters will be there in the sequence?
(1) 4 (2) 12 (3) 16 (4) 17
8. If the first element from the left interchanges place with the tenth element from the left, similarly second with
ninth, third with eighth, fourth with seventh and so on, then which of the following will be seventh to the left of
eighth element from the right?
(1) C (2) D (3) 7 (4) None of these
9. How many such digits are there in the sequence each of which is immediately preceded as well as followed by a
letter?
(1) Nil (2) One (3) Two (4) Three
Direction (Q.10 to Q.14) : Study the following series of alpha-numeric-symbol combination and answer the
questions that follow:
S K 6 £ Q 2 Ré C F 8 E $ G 2 # 4 9 L N 3 U V 5 Y a B 7 W 9
10. How many symbols are there in the above arrangement each of which is immediately preceded by but not
immediately followed by a letter of English alphabet?
(1) Nil (2) One (3) Two (4) Three
11. What should come in place of question mark in the following series?
S9K 6W£ Q72 ? CaF
(1) R Y é (2) é Y C (3) 2 B R (4) R B é
12. If the positions of # and $ are interchanged, so also the positions of £ and é, Q and K and F and V are
interchanged, which of the following will be the ninth element to the left of eighteenth element from the left?
(1) C (2) V (3) 8 (4) é
13. Three of the following four are alike in respect of their positions in the above series. Which is the one that does
not belong to that group?
(1) Q é 8 ( 2) £ R F (3) 6 2 C (4) 2é8
14. S K £ is related to 9 W B in the same way as Q 2 é is related to
(1) a V U (2) a Y 5 (3) 7 B Y (4) a Y V

Q ue. 1 2 3 4 5 6 7 8 9 10 11 12 13 14
Ans. 2 3 3 2 4 3 3 4 4 1 4 1 4 4

www.allenoverseas.com OVERSEAS 129


Class-VII

20.2 Logical Sequence of Words


In this type of questions, certain inter-related words are given and numbered, followed by various sequences of
the numbers denoting them, as alternatives. The candidate is required to arrange these words in a logical
sequence based on a common property and then choose the correctly graded sequence from the given alternatives.
Some common sequences have been discussed below:
1. Sequence of occurrence of events
or various stages in a process
Solved examples
Ex.1 Arrange the following in a meaningful sequence:
(1) Consultation (2) Illness (3) Doctor (4) Treatment
(5) Recovery
(1) 2, 3, 1, 4, 5 (2) 2, 3, 4, 1, 5 (3) 4, 3, 1, 2, 5 (4) 5, 1, 4, 3, 2
Sol. Clearly, illness occurs first. Then one goes to a doctor and after consultation with him, undergoes treatment to
finally attain recovery.
Thus, the correct order is 2, 3, 1, 4, 5. Hence the answer is (1)
Ex.2 Arrange the following in a logical order:
(1) Birth (2) Death (3) Funeral (4) Marriage
(5) Education
(1) 1, 3, 4, 5, 2 (2) 1, 5, 4, 2, 3 (3) 2, 3, 4, 5, 1 (4) 4, 5, 3, 1, 2
Sol. Clearly the given words when arranged in the order of various events as they occur in a man's life, form the
sequence : Birth, Education, Marriage, Death, Funeral.
So the correct order becomes 1, 5, 4, 2, 3
Hence the answer is (2).
2. Sequence of objects in a class
or group, from part to the whole
Solved examples
Ex.3 Arrange the following in a meaningful order, from particular to general :
(1) Family (2) Community (3) Member (4) Locality
(5) Country
(1) 3, 1, 2, 4, 5 (2) 3, 1, 2, 5, 4 (3) 3, 1, 4, 2, 5 (4) 3, 1, 4, 5, 2
Sol. Clearly a member is a part of a family, which in turn is a part of community. The community lives in a locality
which lives within a country.
Thus, the correct order is 3, 1, 2, 4, 5.
Hence the answer is (1).
Ex.4 Arrange the following in a logical order :
(1) Shoulder (2) Wrist (3) Elbow (4) Palm
(5) Finger
(1) 2, 4, 5, 3, 1 (2) 3, 1, 4, 2, 5 (3) 3, 4, 5, 2, 1 (4) 5, 4, 2, 3, 1
Sol. Clearly, we are given the names of parts of a hand, which may be arranged
(1) From top to bottom, i.e., Shoulder, Elbow, Wrist, Palm, Finger, which is 1, 3, 2, 4, 5 or
(2) From bottom to top, i.e., Finger, Palm, Wrist, Elbow, Shoulder, which is 5, 4, 2, 3, 1
Out of these, the sequence 5, 4, 2, 3, 1 is given in the alternatives provided.
Hence the answer is (4).

130 OVERSEAS www.allenoverseas.com


Mental Ability

3. Sequence of increasing / decreasing size,


value, intensity etc.
Solved examples
Ex.5 Arrange the following in a logical sequence from small to big :
(1) Bungalow (2) Flat (3) Cottage (4) House
(5) Palace (6) Mansion
(1) 3, 2, 1, 4, 6, 5 (2) 3, 2, 4, 1, 5, 6 (3) 3, 2, 4, 1, 6, 5 (4) 5, 6, 4, 1, 2, 3
Sol. Clearly, the names of various dwelling units, when arranged in increasing order of their sizes, form the sequence :
Cottage, Flat, House, Bungalow, Mansion, Palace.
Thus, the correct order is 3, 2, 4, 1, 6, 5
Hence the answer is (3).
Ex.6 Arrange the following in a logical order :
(1) Gold (2) Iron (3) Sand (4) Platinum
(5) Diamond
(1) 2, 4, 3, 5, 1 (2) 3, 2, 1, 5, 4 (3) 4, 5, 1, 3, 2 (4) 5, 4, 3, 2, 1
Sol. Clearly, the given names when arranged in order of increasing values, i.e., from cheapest to the most expensive,
form the sequence : Sand, Iron, Gold, Diamond, Platinum.
Thus, the correct order is 3, 2, 1, 5, 4
Hence the answer is (2).

4. Sequence in which a chain of given

objects is formed
Solved examples
Ex.7 Arrange the following in a meaningful sequence :
(1) Phrase (2) Letter (3) Word (4) Sentence
(1) 1, 2, 3, 4 (2) 1, 3, 2, 4 (3) 2, 3, 1, 4 (4) 2, 3, 4, 1
Sol. A group of letters makes a word. A group of words makes a phrase. A group of phrases makes a sentence.
Thus, the correct order is 2, 3, 1, 4
Hence the answer is (3).
Ex.8 Arrange the following in a logical order:
(1) Frog (2) Eagle (3) Grasshopper (4) Snake
(5) Grass
(1) 1, 3, 5, 2, 4 (2) 3, 4, 2, 5, 1 (3) 5, 3, 1, 4, 2 (4) 5, 3, 4, 2, 1
Sol. Clearly, a grasshopper feeds on grass, a frog feeds on a grasshopper, a snake feeds on frog and an eagle feeds
on a snake. Thus, a food-chain is formed.
So, the correct order is 5, 3, 1, 4, 2
Hence the answer is (3).

www.allenoverseas.com OVERSEAS 131


Class-VII

Direction (Q.1 to Q.20) :In each of the following questions, arrange the given words in a meaningful sequence
and then choose the most appropriate sequence from among the alternatives provided below each questions :
1. (1) Honey (2) Flower (3) Bee (4) Wax
(1) 1, 3, 4, 2 (2) 2, 1, 4, 3 (3) 2, 3, 1, 4 (4) 4, 3, 2, 1
2. (1) Police (2) Punishment (3) Crime (4) Justice
(5) Judgement
(1) 1, 2, 3, 4, 5 (2) 3, 1, 2, 4, 5 (3) 3, 1, 4, 5, 2 (4) 5, 4, 3, 2, 1
3. (1) College (2) Child (3) Salary (4) School
(5) Employment
(1) 1, 2, 4, 3, 5 (2) 2, 4, 1, 5, 3 (3) 4, 1, 3, 5, 2 (4) 5, 3, 2, 1, 4
4. (1) Andhra Pradesh (2) Universe (3) Tirupati (4) World
(5) India
(1) 1, 5, 3, 2, 4 (2) 2, 1, 3, 5, 4 (3) 3, 1, 5, 4, 2 (4) 5, 4, 2, 1, 3
5. (1) Atomic Age (2) Metallic Age (3) Stone Age (4) Alloy Age
(1) 1, 3, 4, 2 (2) 2, 3, 1, 4 (3) 3, 2, 4, 1 (4) 4, 3, 2, 1
6. (1) Post-box (2) Letter (3) Envelope (4) Delivery
(5) Clearance
(1) 2, 3, 1, 4, 5 (2) 3, 2, 1, 4, 5 (3) 2, 3, 1, 5, 4 (4) 3, 2, 4, 5, 1
7. (1) Curd (2) Grass (3) Butter (4) Milk
(5) Cow
(1) 2, 5, 4, 1, 3 (2) 4, 2, 5, 3, 1 (3) 5, 2, 3, 4, 1 (4) 5, 2, 4, 1, 3
8. (1) Butterfly (2) Cocoon (3) Egg (4) Worm
(1) 1, 3, 4, 2 (2) 1, 4, 3, 2 (3) 2, 4, 1, 3 (4) 3, 4, 2, 1
9. (1) Study (2) Job (3) Examination (4) Earn
(5) Apply
(1) 1, 3, 2, 5, 4 (2) 1, 2, 3, 4, 5 (3) 1, 3, 5, 2, 4 (4) 1, 3, 5, 4, 2
10. (1) District (2) Village (3) State (4) Town
(5) City
(1) 2, 4, 1, 5, 3 (2) 2, 1, 4, 5, 3 (3) 5, 3, 2, 1, 4 (4) 2, 5, 3, 4, 1
11. (1) Cutting (2) Dish (3) Vegetable (4) Market
(5) Cooking
(1) 1, 2, 4, 5, 3 (2) 3, 2, 5, 1, 4 (3) 4, 3, 1, 5, 2 (4) 5, 3, 2, 1, 4
12. (1) Table (2) Tree (3) Wood (4) Seed
(5) Plant
(1) 1, 2, 3, 4, 5 (2) 1, 3, 2, 4, 5 (3) 4, 5, 2, 3, 1 (4) 4, 5, 3, 2, 1

132 OVERSEAS www.allenoverseas.com


Mental Ability

13. (1) Doctor (2) Fever (3) Prescribe (4) Diagnose


(5) Medicine
(1) 1, 4, 3, 2, 5 (2) 2, 1, 3, 4, 5 (3) 2, 1, 4, 3, 5 (4) 2, 4, 3, 5, 1
14. (1) Hecto (2) Centi (3) Deca (4) Kilo
(5) Deci
(1) 1, 3, 4, 5, 2 (2) 1, 5, 3, 4, 2 (3) 2, 5, 3, 1, 4 (4) 5, 2, 1, 4, 3
15. (1) Yarn (2) Plant (3) Saree (4) Cotton
(5) Cloth
(1) 2, 4, 1, 5, 3 (2) 2, 4, 5, 1, 3 (3) 2, 4, 3, 5, 1 (4) 2, 4, 5, 3, 1
16. 1.Caste 2.Family 3.Newly Married Couple 4.Clan
5.Species
(1) 2,3,1,4,5 (2) 3,4,5,1,2 (3) 3,2,1,4,5 (4) 4,5,3,2,1
17. 1.Presentation 2. Recommendation 3.Arrival 4. Discussion
5. Introduction
(1) 3,5,1,4,2 (2) 3,5,4,2,1 (3) 5,3,1,2,4 (4) 5,3,4,1,2
18. 1. Animal 2. Feline 3. Leapord 4. Mammal
5. Vertebrate 6. Cat
(1) 1,2,3,4,5,6 (2) 1,3,5,4,2,6 (3) 1,4,3,2,5,6 (4) 1,5,4,2,3,6
19. 1. Probation 2. Interview 3. Selection 4. Appointment
5. Advertisment 6. Application
(1) 5,6,2,3,4,1 (2) 5,6,3,2,4,1 (3) 5,6,4,2,3,1 (4) 6,5,4,2,3,1
20. 1. Travel 2. Destination 3. Payment 4. Berth/Seat Number
5. Reservation 6. Availability of Berth/Seat number
(1) 1,2,5,4,3,6 (2) 2,6,3,5,4,1 (3) 5,3,4,1,6,2 (4) 6,2,5,4,3,1

Q u e. 1 2 3 4 5 6 7 8 9 10 11 12 13 14 15
Ans. 3 3 2 3 3 3 4 4 3 1 3 3 3 3 1
Q u e. 16 17 18 19 20
Ans. 3 1 4 1 2

www.allenoverseas.com OVERSEAS 133


Class-VII

Important Notes

134 OVERSEAS www.allenoverseas.com


Mental Ability

CHAPTER 21 SITTING ARRANGEMENT


Ex.1 Seven people - A, B, C, D, E, F and G are standing in a single line, facing North. E is somewhere ahead of F.
There is exactly one person standing between C and G. D is immediately behind A. F is behind both B and D.
If D and C are fourth and fifth in line respectively, then which of the following must be true?
(1) A is first (2) B is first (3) F is sixth (4) F is seventh
Sol. D is immediately behind A i.e.. A D
D and C are fourth and fifth respectively i.e.,
__, __, A, D, C__, __
There is exactly one person between C and G i.e.,

__, __, A D, C__, G.

F is behind B, D and E. So, F is between C and G while the first two position are taken by B and E.

Thus, the sequence becomes:

B/E, B/E, A, D, C, F, G

Clearly, F is sixth in the line. Hence, the answer is (3).

Ex.2 Direction (Q.1 to Q.5): Study the given information carefully and answer the questions that follow:
(i) A, B, C, D, E, F and G are sitting on a wall and all of them are facing East.
(ii) C is to the immediate right of D.
(iii) B is at an extreme end and has E as his neighbour.
(iv) G is between E and F.
(v) D is sitting third from the South end.
1. Who is sitting to the right of E?
(1) A (2) C (3) G (4) F
2. Which of the following pairs of people are sitting at the extreme ends?
(1) A and B (2) A and E (3) C and B (4) F and B
3. Name the person who should change place with C such that he is at the third place from the North end.
(1) E (2) F (3) G (4) D
4. Immediately between which of the following pairs of people is D sitting?
(1) A and C (2) A and F (3) C and E (4) C and F
5. Which of the conditions (i) to (v) given is not required to find out the place in which A is sitting?
(1) (i) (2) (ii) (3) (iii) (4) All are required
Sol. C is to the right of D.
D is third from south. So, B will be at the extreme end from north because it should have E as its neighbour. G
is between E and F. So, the sequence is :

www.allenoverseas.com OVERSEAS 135


Class-VII

F ® East


1. G is sitting to the right of E. Hence, the answer is (3).
2. A and B are sitting at the extreme ends. Hence, the answer is (1).
3. G should change place with C to make it third from north. Hence, the answer is (3).
4. D is sitting between C and F. Hence, the answer is (4).
5. All the statements are required to determine the correct sequence. Hence, the answer is (4).
Ex.3 Directions (Q.1 & Q.2) : Study the following information to answer the given questions:
P, Q, R, S, T, U, V and W are eight friends sitting around a circle facing towards the centre.
(i) W is to the immediate left of P but is not the neighbour of T or S.
(ii) U is to the immediate right of Q and V is the neighbour of T.
(iii) R is between T and U.
1. Which of the following statements is true?
(1) T is between U and Q (2) U is the neighbour of V
(3) V is between W and T (4) W is between P and S
2. What is the position of S?
(1) On the immediate left of Q (2) Second to the right of U
(3) Between Q and U (4) On the immediate left of P
R U
Sol. W is on the immediate left of P. So, anticlockwise, we have: WP.
Q
U is on the immediate right of Q, R is between T and U, V is the T
neighbour of T. So anticlockwise, we have : QURTV.

Since W is not the neighbour of S, so S does not sit between V S


V
and W. So, S sits between P and Q.

Thus, the seating arrangement is as shown. W P

1. T is between R and V : U is the neighbour of Q and R ; V is between W and T; W is between V and P. Hence,
the answer is (3).

2. S's position is

(1) to the immediate left of Q (2) second to the left of U

(3) between Q and P (4) to the immediate right of P

Hence, the answer is (1).

136 OVERSEAS www.allenoverseas.com


Mental Ability

1. Five children are Sitting in a row. S is sitting next to P but not T. K is sitting next to R who is sitting to the extreme
left and T is not sitting next to K. Who are sitting adjacent to S?
(1) K and P (2) R and P (3) P and S (4) P and T
2. In the Olympic game, the flags of six nations were flown on the masts in the following way:
The flag of America was to the left of Indian tricolour and to the right of the flag of France The flag of Australia
was to the right of the Indian flag but was to the left of the flag of Japan, which was to the left of the flag of
China. Find the two flags which are in the centre.
(1) India & Australia (2) America & India
(3) Japan & Australia (4) America & Australia
Directions (Q.3 to Q.5): Study the given information carefully and answer the questions that follow:
(i) There are seven books one each on Psychology, Hindi, English, Sociology, Economics. Education and
Accountancy, lying on a table one above the other.
(ii) Sociology is on the top of all the books.
(iii) Accountancy is immediately below Education which is immediately below Sociology.
(iv) Economics is immediately above Psychology but not in the middle.
(v) Hindi is immediately below Psychology.
3. Economics is between which of the following books?
(1) Accountancy and Education (2) Psychology and Hindi
(3) English and Psychology (4) Psychology and Sociology
4. Which three books are between Accountancy and Hindi?
(1) English, Economics and Psychology (2) Economics, Psychology, Sociology
(3) Economics, Psychology and Hindi (4) Cannot be determined
5. If Sociology and English, Accountancy and Hindi, Education and Psychology interchange their positions, which
book will be between Psychology and Sociology?
(1) Accountancy (2) Psychology (3) Hindi (4) Economics
Directions (Q.6 & Q.7): Read the following information to answer the given questions.
Seven children A, B, C, D, E, F and G are standing in a line. G is to the right of D and to the left of B. A is to the
right of C. A and D have one child between them. E & B and F & D have two children between them.
6. Who is to the extreme right?
(1) B (2) E (3) F (4) G
7. Who is exactly in the middle?
(1) A (2) D (3) F (4) G
Directions (Q.8 to Q.10) : Study the following information to answer the given questions.
(i) Eight friends A, B, C, D, E, F, G and H are seated in a circle facing centre.
(ii) D is between B and G and F is between A and H.
(iii) E is second to the right of A.

www.allenoverseas.com OVERSEAS 137


Class-VII

8. Which of the following is A's position?


(1) Left of F (2) Right of F (3) Between E and F (4) Can't be determined
9. Which of the informations statement are not required to ascertain the position of C?
(1) (i) above (2) Either (ii) or (iii) above (3) (iii) above (4) All are required
10. Which of the following is C's position?
(1) Between E and F (2) Between G and E (3) Second to the left of B (4) None of these
11. Eight books are kept one over the other. Counting from the top, the second, fifth and sixth books are on Plays.
Two books on Plays are between two books on Composition. One books of Plays is between two books on poetry
while the book at the top of the book of Literature is a book of Composition Which book is fourth from the top?
(1) Plays (2) Poetry (3) Composition (4) Literature
Directions (Q.12 to Q.14) : P, Q, R, S, T, U, V and W are sitting round the circle and facing the centre:
i. P is second to the right of T who is the neighbour of R and V.
ii S is not the neighbour of U.
iii. V is the neighbour of U.
iv. Q is not between S and W. W is between Q and S.
12. What is the position of S?
(1) Between U and V (2) Second to the right of P
(3) To the immediate right of P (4) Data inadequate
13. Which two of the following are not neighbours?
(1) R and V (2) U and V (3) R and P (4) Q and W
14. Which of the following is correct?
(1) P is to the immediate right of Q (2) R is between U and V
(3) Q is to the immediate right of W (4) U is between W and S
Direction (Q. 15 to Q.17) : Read the following information carefully to answer the questions given below:
Nine cricket fans are watching a match in a stadium. Seated in one row, they are J, K, L, M, N, O, P, Q and R.
L is to the right of M and at third place to the right of N. K is at one end of the row. Q is seated adjacent to both
O and P. O is at the third place to the left of K. J is immediate next to left of O.
15. Who is sitting in the centre of the row?
(1) L (2) J (3) O (4) KO
16. Who is at the other end of the row?
(1) J (2) N (3) P (4) R
17. Which of the following statement is true?
(1) N is two seats away from J. (2) M is at one extreme end.
(3) R and P are neighbours. (4) There is one person between L and O.
Directions (Q.18 to Q. 21) : Read the following information carefully and answer the questions given below:
A, B, C, D, E, F and G are seven students in a class. They are sitting on three benches I, II and III in such a way
that there are at least two of them on each bench and there is at least one girl on each bench. C, a girl student,
does not Sit with A, E and D. F, a boy student, sits with only B. A sits with his best friend in bench I. G sits on
bench III. E is brother of C.
18. On which bench do three students sit?
(1) I (2) II (3) III (4) I or II

138 OVERSEAS www.allenoverseas.com


Mental Ability

19. How many girl students are there?


(1) 3 (2) 4 (3) 3 or 4 (4) Data inadequate
20. Who sits with C?
(1) B (2) D (3) E (4) G
21. Which of the following is a group of girls?
(1) BAC (2) BCD (3) BFC (4) CDF
Direction (Q.22 to Q.25): Read the following information and answer the question that follow:
Manoj and Naveen play Hockey and Volleyball. Vinod and Manoj play Baseball and Hockey. Prashant and
Naveen play Volleyball and Cricket. Vinod, Prashant and Deepak play Baseball and Hockey.
22. Who plays largest number of games?
(1) Prashant (2) Deepak (3) Manoj (4) Vinod
23. Who plays Hockey, Baseball and Volleyball only?
(1) Prashant (2) Manoj (3) Deepak (4) Naveen
24. Who plays Volleyball, Cricket, and Hockey only?
(1) Manoj (2) Deepak (3) Prashant (4) Naveen
25. Who plays Baseball, Cricket and Volleyball?
(1) Prashant (2) Manoj (3) Deepak (4) Naveen
Direction (Q.26 to Q.30) : Read the following information and answer the question that follow:
(A) Eleven students A ,B ,C ,D ,E ,F ,G ,H ,I ,J and K are sitting in the first row of the class facing the teacher.
(B) D who is to the immediate left of F is 2 nd to the right of C.
(C) A is 2nd to the right of E ,who is at one of the ends.
(D) J is the immediate neighbour of A and B and third to the left of G.
(E) H is to the immediate left of D and third to the right of I.
26. Who is sitting in the middle of the row?
(1) B (2) C (3) G (4) I
27. Which of the following group is sitting to the right of G?
(1) CHDE (2) CHDF (3) IBJA (4) ICHDF
28. Which of the following statments is true in the context of the above sitting arrangment?
(1) There are three students sitting between D and G.
(2) K is between A and J.
(3) B is sitting between J and I.
(4) G and C are neighbours sitting to the immediate right of H.
29. In the above sitting arrangment which of the following staments is superfluous?
(1) A (2) B (3) D (4) None
30. If E and D, C and B , A and H and K and F interchange their positions , which of the following pairs of students
is sitting at the ends?
(1) D and E (2) D and K (3) K and F (4) D and A

Que. 1 2 3 4 5 6 7 8 9 10 11 12 13 14 15
Ans. 1 1 3 1 3 3 2 2 4 4 3 3 1 3 2
Que. 16 17 18 19 20 21 22 23 24 25 26 27 28 29 30
Ans. 2 4 1 1 4 2 1 2 4 1 4 2 3 4 2

www.allenoverseas.com OVERSEAS 139


Class-VII

Important Notes

140 OVERSEAS www.allenoverseas.com


Mental Ability

CHAPTER 22 DOT SITUATION

In such type of problems one or more dot(s) is (are) placed somewhere in region enclosed in between
geometrical figures such as triangle, square, rectangle, circle etc.
A candidate is required to identify one alternative figure which has the same region(s) as the region(s) marked
by dot(s) in the given figure.
Solved examples
Ex. Select a figure from the four options, that has the same conditions of the placement of the dots as in figure (X).

(X)

(1) (2) (3) (4)

Sol. We require a figure amongst the four options that has


1. a region common to all the three simple figures and
2. a region that lies in circle but not in triangle and square
Such two conditions are available in option (2) only.
Let us illustrate it with the diagram.

Hence, the answer is (2).

Direction (Q.1 to Q.15) : In each of the following question, from amongst the figures marked (1), (2), (3) and
(4), select the one which satisfies the same condition of placement of the dot(s) as in the given figure (x)

1.

(X)

(1) (2) (3) (4)

www.allenoverseas.com OVERSEAS 141


Class-VII

2.

(X)

(1) (2) (3) (4)

3.

(X)

(1) (2) (3) (4)

4.

(X)

(1) (2) (3) (4)

5.

(X)

(1) (2) (3) (4)

6.

(X)

(1) (2) (3) (4)

142 OVERSEAS www.allenoverseas.com


Mental Ability

7.

(X)

(1) (2) (3) (4)

8.

(X)

(1) (2) (3) (4)

9.

(X)

(1) (2) (3) (4)

10.

(X)

(1) (2) (3) (4)

11.

(X)

(1) (2) (3) (4)

www.allenoverseas.com OVERSEAS 143


Class-VII

12.

(X)

(1) (2) (3) (4)

13.

(X)

(1) (2) (3) (4)

14.
(X)

(1) (2) (3) (4)

15.

(X)

(1) (2) (3) (4)

Que. 1 2 3 4 5 6 7 8 9 10 11 12 13 14 15
Ans. 2 1 3 2 4 4 1 1 2 1 3 4 4 2 1

144 OVERSEAS www.allenoverseas.com


Mental Ability

SPOTTING OUT
CHAPTER 23 THE EMBEDDED FIGURES
The Embedded Figure Test (EFT) is designed to measure disembedding as a restructuring skill, which results
from the use of style. In this test, students are required to discern simple geometric figures from more complicated
patterns. This test measures the general intelligence or some specific ability of the person.

Solved examples
You are given a figure (X) followed by four alternatives (1), (2), (3) and (4) such that figure (X) is embedded in one
of them. Candidates have to identify the figure which contains figure (X).
Ex.1 Find out the alternative figure which contains figure (X).

(X) (1) (2) (3) (4)

Sol.
Explanation

Hence, the answer is (1).


Ex.2 Find out the alternative figure which contains figure (X) as its part

(X) (1) (2) (3) (4)

Sol.

Hence, the answer is (4).

www.allenoverseas.com OVERSEAS 145


Class-VII

Directions (Q.1 to Q.6): In each of the following questions, you are given a figure (X) followed by four
alternative figures (1), (2), (3) and (4) such that fig. (X) is embedded in one of them. Trace out the figure which
contains fig. (X) as its part.

1.

(X) (1) (2) (3) (4)

2.

(X) (1) (2) (3) (4)

3.

(1) (2) (3) (4)


(X)

4.

(X) (1) (2) (3) (4)

5.

(X) (1) (2) (3) (4)

6.

(X) (1) (2) (3) (4)

Directions (Q.7 to Q.13): In each of the following questions, you are given a fig. (X) followed by four alternative
figures (1), (2), (3) and (4) such that fig. (X) is embedded in one of them. Trace out the alternative figure which
contains fig. (X) as its part.

7.

(X) (1) (2) (3) (4)

146 OVERSEAS www.allenoverseas.com


Mental Ability

8.

(X) (1) (2) (3) (4)

9.

(X) (1) (2) (3) (4)

10.

(X) (1) (2) (3) (4)

11.

(X) (1) (2) (3) (4)

12.

(X) (1) (2) (3) (4)

13.

(X) (1) (2) (3) (4)


Directions (Q.14 to Q.20): In each of the following questions, you are given a figure (X) followed by four
alternative figures (1), (2), (3) and (4) such that fig (X) is embeded in one of them. Trace out the alternative figure
which contains fig (X) as is part.

14.

(X) (1) (2) (3) (4)

15.

(X) (1) (2) (3) (4)

www.allenoverseas.com OVERSEAS 147


Class-VII

16.

(X) (1) (2) (3) (4)

17.

(X) (1) (2) (3) (4)

18.

(X) (1) (2) (3) (4)

19.

(X) (1) (2) (3) (4)

20.

(X) (1) (2) (3) (4)

Que. 1 2 3 4 5 6 7 8 9 10
Ans. 3 3 4 4 4 2 4 1 4 3
Que. 11 12 13 14 15 16 17 18 19 20
Ans. 4 2 2 4 4 4 3 2 4 4

148 OVERSEAS www.allenoverseas.com


Mental Ability

COMPLETION OF
CHAPTER 24 INCOMPLETE PATTERN
In this type of problems, a figure or a matrix, containing a set of figures following a particular sequence or
pattern is given, in which a part, generally a quarter, is left blank. This problem figure is followed by four alternative
figures. The candidate is required to select the one which best fits into the blank space of problem figure so
as to complete the original pattern.
Solved examples
Ex.1 Select a figure from amongst the four alternatives, which when placed in the blank space of fig. (X) would
complete the pattern.

?
(X) (1) (2) (3) (4)

Sol. Clearly, fig. (2) will complete the pattern when placed in the blank space of fig. (X) as shown below :

Hence, the answer is (2).


Ex.2 Complete the pattern in fig. (X) by selecting one of the figures from the four alternatives.

(X) (1) (2) (3) (4)

Sol. Clearly, fig. (1) will complete the pattern when placed in the blank space of fig. (X) as shown below :

Hence, the answer is (1).

www.allenoverseas.com OVERSEAS 149


Class-VII

Directions (Q.1 to Q.20) : In each of the following questions, select a figure from amongst the four alternatives,
which when placed in the blank space of fig. (X) would complete the pattern.

1. ?
(X) (1) (2) (3) (4)

2. ?

(X) (1) (2) (3) (4)

3. ?

(X) (1) (2) (3) (4)

4.

(X) (1) (2) (3) (4)

5.
?

(X) (1) (2) (3) (4)

6.
?
(X) (1) (2) (3) (4)

7.
?
(X) (1) (2) (3) (4)

150 OVERSEAS www.allenoverseas.com


Mental Ability

8. ?

(X) (1) (2) (3) (4)

9.

(X) (1) (2) (3) (4)

10. ?
(X) (1) (2) (3) (4)

11. ?
(X) (1) (2) (3) (4)

12.
?
(X) (1) (2) (3) (4)

?
13.

(X) (1) (2) (3) (4)

14. ?
(X) (1) (2) (3) (4)

?
15.

(X) (1) (2) (3) (4)

www.allenoverseas.com OVERSEAS 151


Class-VII

16. ?
(X) (1) (2) (3) (4)

17.

(X) (1) (2) (3) (4)

18. ?
(X) (1) (2) (3) (4)

19.

(?)

(X) (1) (2) (3) (4)

20.

?
(X) (1) (2) (3) (4)

Que. 1 2 3 4 5 6 7 8 9 10 11 12 13 14 15 16 17 18 19 20
A ns. 3 4 2 4 3 3 1 1 4 1 2 1 2 2 3 4 2 3 1 4

152 OVERSEAS www.allenoverseas.com


Mental Ability

CHAPTER 25 FIGURE MATRIX


In such type of problems a 2×2 or 3×3 grid is given. This grid has some design or symbols to form a pattern. But
a cell of the grid is left empty. A candidate requires to fill up the cell. Now, one needs to analyse the grid and
identify a rule along row-wise or column-wise in the grid.
Solved example
Ex. Complete the given pattern.

?
(1) (2) (3) (4)

Sol. Let us consider horizontally. The second figure is obtained from the first figure by moving the line segment to
the opposite side of the square boundary and replacing it with two similar line segments. Also, the element in
the lower-left corner gets replaced by two similar elements – one placed in the upper-left and the other
placed in the lower right corner. Hence, the answer is (1).

Direction (Q.1 to Q.15) : In each of the following question, find out which of the answer figures (1), (2),
(3) and (4) completes the given figure matrix?

1.

(1) (2) (3) (4)

2.

(1) (2) (3) (4)

www.allenoverseas.com OVERSEAS 153


Class-VII

3.

(1) (2) (3) (4)

4.

(1) (2) (3) (4)

5.
?

(1) (2) (3) (4)

6.

(1) (2) (3) (4)

7.

(1) (2) (3) (4)

154 OVERSEAS www.allenoverseas.com


Mental Ability

8.

(1) (2) (3) (4)

9.

(1) (2) (3) (4)

10.

(1) (2) (3) (4)

11.

(1) (2) (3) (4)

www.allenoverseas.com OVERSEAS 155


Class-VII

12.

(1) (2) (3) (4)

13.

?
(1) (2) (3) (4)

14.

?
(1) (2) (3) (4)

15.

?
(1) (2) (3) (4)

Q u e. 1 2 3 4 5 6 7 8 9 10 11 12 13 14 15
Ans. 3 1 4 3 2 4 1 2 1 2 2 1 4 2 4

156 OVERSEAS www.allenoverseas.com

You might also like